78
Optimizing Oral Antiplatelet Therapy in Acute Coronary Syndrome Presented as a Midday Symposium at the 45 th ASHP Midyear Clinical Meeting and Exhibition Tuesday, December 7, 2010 Anaheim, California Planned and conducted by ASHP Advantage and supported by an educational grant from AstraZeneca

Optimizing Oral Antiplatelet Therapy in Acute Coronary ...Optimizing Oral Antiplatelet Therapy in Acute Coronary Syndrome Toby C. Trujillo, Pharm.D., BCPS (AQ Cardiology) PRESENTATION

  • Upload
    others

  • View
    4

  • Download
    0

Embed Size (px)

Citation preview

Page 1: Optimizing Oral Antiplatelet Therapy in Acute Coronary ...Optimizing Oral Antiplatelet Therapy in Acute Coronary Syndrome Toby C. Trujillo, Pharm.D., BCPS (AQ Cardiology) PRESENTATION

Optimizing Oral Antiplatelet Therapy in Acute Coronary Syndrome

Presented as a Midday Symposium at the 45

th ASHP Midyear Clinical Meeting and Exhibition

Tuesday, December 7, 2010

Anaheim, California

Planned and conducted by ASHP Advantage and supported by an educational grant from AstraZeneca

Page 2: Optimizing Oral Antiplatelet Therapy in Acute Coronary ...Optimizing Oral Antiplatelet Therapy in Acute Coronary Syndrome Toby C. Trujillo, Pharm.D., BCPS (AQ Cardiology) PRESENTATION

Please be advised that this activity is being audio recorded for archival purposes and, in some cases, for repurposing of the content for enduring materials.

2

Page 3: Optimizing Oral Antiplatelet Therapy in Acute Coronary ...Optimizing Oral Antiplatelet Therapy in Acute Coronary Syndrome Toby C. Trujillo, Pharm.D., BCPS (AQ Cardiology) PRESENTATION

Optimizing Oral Antiplatelet Therapy in Acute Coronary Syndrome

A G E N D A

11:30 a.m. – 11:35 a.m. Introductory Remarks Paul P. Dobesh, Pharm.D., FCCP, BCPS (AQ-Cardiology) 11:35 a.m. – 12:00 p.m. Update on Oral Antiplatelet Therapy in Acute Coronary

Syndrome Toby C. Trujillo, Pharm.D., BCPS (AQ-Cardiology) 12:00 p.m. – 12:40 p.m. Clinical Challenges in Oral Antiplatelet Therapy Julie H. Oestreich, Pharm.D., Ph.D. 12:40 p.m. – 1:15 p.m. Emerging P2Y12 Treatment Strategies Paul P. Dobesh, Pharm.D., FCCP, BCPS (AQ-Cardiology) 1:15 p.m. – 1:30 p.m. Faculty Discussion and Audience Questions All Faculty

F A C U L T Y

Paul P. Dobesh, Pharm.D., FCCP, BCPS (AQ Cardiology) Activity Chair and Moderator Associate Professor of Pharmacy Practice College of Pharmacy University of Nebraska Medical Center Omaha, Nebraska

Julie H. Oestreich, Pharm.D., Ph.D. Assistant Professor of Pharmacy Practice College of Pharmacy University of Nebraska Medical Center Omaha, Nebraska

Toby C. Trujillo, Pharm.D., BCPS (AQ Cardiology) Associate Professor University of Colorado Denver School of Pharmacy Clinical Specialist – Cardiology/Anticoagulation University of Colorado Hospital Aurora, Colorado

3

Page 4: Optimizing Oral Antiplatelet Therapy in Acute Coronary ...Optimizing Oral Antiplatelet Therapy in Acute Coronary Syndrome Toby C. Trujillo, Pharm.D., BCPS (AQ Cardiology) PRESENTATION

Optimizing Oral Antiplatelet Therapy in Acute Coronary Syndrome

D I S C L O S U R E S T A T E M E N T

In accordance with the Accreditation Council for Continuing Medical Education’s Standards for Commercial Support and the Accreditation Council for Pharmacy Education’s Guidelines for Standards for Commercial Support, ASHP Advantage requires that all individuals involved in the development of activity content disclose their relevant financial relationships. A person has a relevant financial relationship if the individual or his or her spouse/partner has a financial relationship (e.g., employee, consultant, research grant recipient, speakers bureau, or stockholder) in any amount occurring in the last 12 months with a commercial interest whose products or services may be discussed in the educational activity content over which the individual has control. The existence of these relationships is provided for the information of participants and should not be assumed to have an adverse impact on presentations. All faculty and planners for ASHP Advantage education activities are qualified and selected by ASHP Advantage and required to disclose any relevant financial relationships with commercial interests. ASHP Advantage identifies and resolves conflicts of interest prior to an individual’s participation in development of content for an educational activity. The faculty and planners report the following relationships: Paul P. Dobesh, Pharm.D., FCCP, BCPS (AQ Cardiology)

Dr. Dobesh declares that he has served as a consultant to sanofi-aventis and has received research grants from AstraZeneca and Eli Lilly and Company. Julie H. Oestreich, Pharm.D., Ph.D.

Dr. Oestreich declares that she has no relationships pertinent to this activity. Toby C. Trujillo, Pharm.D., BCPS (AQ Cardiology)

Dr. Trujillo declares that he has served as a speaker for sanofi-aventis. Carla J. Brink, M.S., B.S.Pharm.

Ms. Brink declares that she has no relationships pertinent to this activity.

4

Page 5: Optimizing Oral Antiplatelet Therapy in Acute Coronary ...Optimizing Oral Antiplatelet Therapy in Acute Coronary Syndrome Toby C. Trujillo, Pharm.D., BCPS (AQ Cardiology) PRESENTATION

Optimizing Oral Antiplatelet Therapy in Acute Coronary Syndrome

A C T I V I T Y O V E R V I E W

Despite recent advances in the treatment of acute coronary syndrome (ACS), mortality associated with both ST-segment elevation myocardial infarction (STEMI) and non-ST-segment elevation myocardial infarction (NSTEMI) remains unacceptably high. Patients who experience an acute myocardial infarction are at increased risk for future coronary episodes, and mortality increases with subsequent events. While the need for dual antiplatelet therapy in patients with ACS is well established, pharmacists are met with many clinical dilemmas as they seek to optimize therapy. In this symposium, the faculty will tackle many of these questions, including duration of oral antiplatelet therapy, clopidogrel nonresponsiveness, drug interactions with proton pump inhibitors, and laboratory testing of platelet reactivity and patient genotypes. In addition, emerging treatment strategies, including triple antiplatelet therapy and pipeline antiplatelet agents, will be addressed. Using patient case examples and interactive questions, the faculty will engage participants in the clinical decision-making process involved in optimizing oral antiplatelet therapy for patients with ACS. A C T I V I T Y O B J E C T I V E S

At the conclusion of this knowledge-based educational activity, participants should be able to

Identify patient factors to consider when selecting an appropriate oral antiplatelet therapy.

Describe the implications of recent clinical trials evaluating the safety and efficacy of current and emerging oral antiplatelet therapies for patients with acute coronary syndrome.

Define clopidogrel nonresponsiveness and devise a treatment strategy to manage it.

List factors to consider when deciding whether to use laboratory testing of platelet reactivity or patient genotype to guide selection of oral antiplatelet therapy.

L I S T O F A B B R E V I A T I O N S

For a list of abbreviations used in this symposium, please see page 76.

5

Page 6: Optimizing Oral Antiplatelet Therapy in Acute Coronary ...Optimizing Oral Antiplatelet Therapy in Acute Coronary Syndrome Toby C. Trujillo, Pharm.D., BCPS (AQ Cardiology) PRESENTATION

Optimizing Oral Antiplatelet Therapy in Acute Coronary Syndrome

C O N T I N U I N G E D U C A T I O N A C C R E D I T A T I O N

The American Society of Health-System Pharmacists is accredited by the Accreditation Council for Pharmacy Education as a provider of continuing pharmacy education. This activity provides 2 hours (0.2 CEUs) of continuing pharmacy education credit (ACPE activity #204-000-10-480-L01P).

Attendees must complete a Continuing Pharmacy Education Request online and may immediately print their official statements of continuing pharmacy education credit at the ASHP Learning Center at http://ce.ashp.org following the activity. Complete instructions for receiving your statement of continuing pharmacy education online are on the next page. Be sure to record the session code beginning with “A” announced during the activity.

Your educational opportunities related to antiplatelet therapy in patients with ACS reach beyond today’s symposium…

A live webinar on March 2, 2011, where faculty will explore issues raised by participant questions in today’s symposium (1 hour CPE). Informational podcast interviews with the faculty about their experiences managing patients with ACS. e-Newsletters featuring tips for incorporating information related to the use of oral antiplatelet therapy in ACS into practice, as well as updates on emerging information. Web-based activity based on today’s live symposium (2 hours CPE, but please note that individuals who claim CPE credit for the live symposium are ineligible to claim credit for the web-based activity).

For more information and to sign up to receive e-mail updates about this educational series, go to

www.ashpadvantage.com/optimize

6

Page 7: Optimizing Oral Antiplatelet Therapy in Acute Coronary ...Optimizing Oral Antiplatelet Therapy in Acute Coronary Syndrome Toby C. Trujillo, Pharm.D., BCPS (AQ Cardiology) PRESENTATION

Optimizing Oral Antiplatelet Therapy in Acute Coronary Syndrome

Instructions for Processing CPE online at http://ce.ashp.org

The ASHP Learning Center allows participants to obtain statements of continuing pharmacy education (CPE) conveniently and immediately using any computer with an internet connection. To obtain your CPE statements for ASHP Advantage activities, please visit

http://ce.ashp.org 1. Log in to the ASHP Learning Center using your e-mail address and password.

If you have not logged in to the new ASHP Learning Center (launched August 2008) and are not a member of ASHP, you will need to set up an account by clicking on “Become a user” and following the instructions.

2. Once logged in to the site, click on “Process Meeting CE.”

3. If you are a registered attendee at the ASHP Midyear Clinical Meeting, click on the start button to the right of ASHP Midyear Clinical Meeting 2010.

If you are not registered to attend the ASHP Midyear Clinical Meeting, click on the start link to the right of the activity title. If this activity title does not appear in your meeting list, enter the 5-digit activity code in the box above the list and click submit. The activity code for this activity is 10480. Click register again when prompted. When you receive the “thank you for registering” message, click continue. This step will bring you back to your meeting list. Click on the start link to the right of the activity title. Do not click on “remove" or you will not be able to process CE for this activity.

4. Click on the click here link to view sessions associated with the day of the activity. This activity was held on Tuesday, December 7, 2010.

5. Enter the session code (e.g., A12345 and note that the letter is case sensitive) that was announced during the activity, and select the number of hours equal to your participation in the activity.

6. Click submit to receive the attestation page.

7. Confirm your participation and click submit.

8. New this year, complete the overall Midyear evaluation and click the “finish” button. You will then be able to view and print your transcript.

Date of Activity Activity Code Session Code

(announced during the live activity)

CPE credit hours

Tuesday

December 7, 2010

10480 2

NEED HELP? Contact ASHP Advantage at [email protected].

7

Page 8: Optimizing Oral Antiplatelet Therapy in Acute Coronary ...Optimizing Oral Antiplatelet Therapy in Acute Coronary Syndrome Toby C. Trujillo, Pharm.D., BCPS (AQ Cardiology) PRESENTATION

Optimizing Oral Antiplatelet Therapy in Acute Coronary Syndrome

8

Page 9: Optimizing Oral Antiplatelet Therapy in Acute Coronary ...Optimizing Oral Antiplatelet Therapy in Acute Coronary Syndrome Toby C. Trujillo, Pharm.D., BCPS (AQ Cardiology) PRESENTATION

Optimizing Oral Antiplatelet Therapy in Acute Coronary Syndrome

Toby C. Trujillo, Pharm.D., BCPS (AQ Cardiology) Associate Professor University of Colorado Denver School of Pharmacy Clinical Specialist – Cardiology/Anticoagulation University of Colorado Hospital Aurora, Colorado Toby C. Trujillo, Pharm.D., BCPS (AQ Cardiology), is Associate Professor of Clinical Pharmacy at the University of Colorado Denver School of Pharmacy in Aurora, Colorado. He also serves as Clinical Specialist in cardiovascular pharmacotherapy and anticoagulation at University of Colorado Hospital. Dr. Trujillo earned his Bachelor of Science degree in biochemistry from the University of California, Davis and his Doctor of Pharmacy degree from the University of California, San Francisco, where he also completed a residency in pharmacy practice. Dr. Trujillo completed a fellowship in cardiovascular pharmacotherapy at The University of Arizona, and he is a board-certified pharmacotherapy specialist with added qualifications in cardiology. Dr. Trujillo’s current responsibilities at University of Colorado Hospital include providing clinical pharmacy services to cardiology, as well as directing the inpatient anticoagulation program. He also serves as co-chair of the anticoagulation subcommittee of the P&T committee. Dr. Trujillo currently serves as a preceptor to both pharmacy students and residents. His lectures at the School of Pharmacy focus on ischemic heart disease, antithrombotic therapy, and other cardiology and critical care topics. Within the American Heart Association, Dr. Trujillo is a member of the Clinical Pharmacology Committee, which resides under the Council on Clinical Cardiology. He has also served a number of committees within the American College of Clinical Pharmacy, served as a speaker on numerous occasions on a national level, and authored several articles and book chapters in the area of cardiovascular pharmacotherapy. Dr. Trujillo is a member of the American Society of Health-System Pharmacists, and he helped develop current standards for postgraduate year 2 (PGY2) cardiology residency training programs.

9

Page 10: Optimizing Oral Antiplatelet Therapy in Acute Coronary ...Optimizing Oral Antiplatelet Therapy in Acute Coronary Syndrome Toby C. Trujillo, Pharm.D., BCPS (AQ Cardiology) PRESENTATION

Optimizing Oral Antiplatelet Therapy in Acute Coronary Syndrome

Toby C. Trujillo, Pharm.D., BCPS (AQ Cardiology) P R E S E N T A T I O N

Update on Oral Antiplatelet Therapy in Acute Coronary Syndrome O V E R V I E W

Antiplatelet therapy has been a cornerstone in the treatment of patients with acute coronary syndrome (ACS) since the 1980s. Depending on the ACS subpopulation being studied, the implementation of aspirin (ASA) monotherapy provided significant reductions in morbidity and mortality. Although the routine use of ASA became the standard of care, a significant number of patients continued to suffer from major ischemic events. The development of ticlopidine and clopidogrel in the 1990s represented another major advance in the treatment of cardiovascular disease. Studies quickly established that at a minimum these agents were equivalent and sometimes superior to ASA monotherapy in the prevention of ischemic events in multiple patient populations with atherosclerotic vascular disease, including ACS. Interest in monotherapy for ACS waned with the publication of the CURE trial, which provided the initial support for dual oral antiplatelet therapy. Along with the dramatic increase in the use of stents in patients undergoing percutaneous coronary intervention (PCI), clinical trials have established the combination of ASA pus clopidogrel as standard of care in patients with ACS over the last decade. Similar to the earlier situation with ASA monotherapy, however, substantial numbers of patients continue to experience ischemic events on dual antiplatelet therapy. In addition, there has been a growing recognition that variable response to clopidogrel may contribute to the residual clinical burden seen with dual antiplatelet therapy. Prasugrel is a new antiplatelet agent with a mechanism of action similar to clopidogrel that may offer pharmacokinetic and pharmacodynamic advantages. In a head-to-head comparison in patients with ACS, prasugrel was more effective at reducing myocardial infarction than clopidogrel when both were added to ASA therapy. The impact of dual antiplatelet therapy will continue to be modified with the development of new oral antiplatelet agents, such as prasugrel.

10

Page 11: Optimizing Oral Antiplatelet Therapy in Acute Coronary ...Optimizing Oral Antiplatelet Therapy in Acute Coronary Syndrome Toby C. Trujillo, Pharm.D., BCPS (AQ Cardiology) PRESENTATION

Update on Oral Antiplatelet Therapy in Acute Coronary Syndrome

Toby C. Trujillo, Pharm.D., BCPS (AQ Cardiology)Associate Professor

University of Colorado Denver School of PharmacyAurora, Colorado

UnderstandingAcute Coronary Syndrome (ACS)

Non-ST Elevation(UA/NSTEMI)

ST Elevation(STEMI)

FibrinolysisPCI +/-Stent

MedicalTherapy

UA = unstable angina, NSTEMI = non-ST-segment elevation myocardial infarction, STEMI = ST-segment elevation myocardial infarction; PCI = percutaneous coronary intervention

ACC/AHA Guidelines•• NSTE ACSNSTE ACS

–– Early invasive for high riskEarly invasive for high risk–– --blockersblockers–– NitratesNitrates–– ACE inhibitors/ARBACE inhibitors/ARB–– Aldosterone antagonistsAldosterone antagonists

StatinsStatins

•• STEMISTEMI–– Reperfusion therapyReperfusion therapy

•• Primary PCIPrimary PCI•• FibrinolyticsFibrinolytics

–– --blockersblockers–– Nitrates Nitrates –– ACE inhibitors/ARBACE inhibitors/ARB

–– StatinsStatins–– AnticoagulantsAnticoagulants–– Antiplatelet therapyAntiplatelet therapy

•• Aspirin (ASA)Aspirin (ASA)•• P2YP2Y1212 receptor antagonistsreceptor antagonists•• GP IIb/IIIa inhibitorsGP IIb/IIIa inhibitors

–– Aldosterone antagonistsAldosterone antagonists–– StatinsStatins–– Anticoagulants Anticoagulants –– Antiplatelet therapyAntiplatelet therapy

•• AspirinAspirin•• P2YP2Y1212 receptor antagonistsreceptor antagonists•• GP IIb/IIIa inhibitorsGP IIb/IIIa inhibitors

Anderson JL et al. J Am Coll Cardiol. 2007; 50:e1-e157.King SB 3rd et al. J Am Coll Cardiol. 2008; 51:172-209.

11

Page 12: Optimizing Oral Antiplatelet Therapy in Acute Coronary ...Optimizing Oral Antiplatelet Therapy in Acute Coronary Syndrome Toby C. Trujillo, Pharm.D., BCPS (AQ Cardiology) PRESENTATION

Existing and Future Antiplatelet Agents

Angiolillo DJ et al. Eur Heart J. 2010; 31:17-28.

17.120

Unstable Angina

2530

3.34

11.8

Acute Myocardial Infarction

Aspirin in Acute Coronary Syndrome

*p<0.0001

Death or Ml

*p=0.003

Reocclusion

*p=0.012

Ml

*p<0.011

Death

6.5

Placebo ASA0

5

10

15

% o

f P

atie

nts

11

Placebo ASA0

10

20

1.9

Placebo ASA0

1

2

39.4

5

10

1990; 336:827-30. 1992; 19:671-7. 1988; 2:349-60.

Placebo ASA0

RISC Group. Lancet Roux et al. JACC ISIS-2. Lancet ISIS-2. Lancet1988; 2:349-60.

CURE StudyCURE StudyPrimary Endpoint: MI/Stroke/CV DeathPrimary Endpoint: MI/Stroke/CV Death

0.080.08

0.100.10

0.120.12

0.140.14

azar

d R

ate

azar

d R

ate

ClopidogrelClopidogrel

Placebo Placebo + ASA+ ASA

0 000090 00009

20%20%RRRRRR

0.000.00

0.020.02

0.040.04

0.060.06

Cu

mu

lati

ve H

Cu

mu

lati

ve H

Clopidogrel Clopidogrel + ASA+ ASA

33 66 99Months of FollowMonths of Follow--upup

pp = 0.00009= 0.00009n = 12,562n = 12,562

00 1212

Yusuf S et al. N Engl J Med. 2001; 345:494-502.

RRR = relative risk reduction

Enlarged slide on page 21

12

Page 13: Optimizing Oral Antiplatelet Therapy in Acute Coronary ...Optimizing Oral Antiplatelet Therapy in Acute Coronary Syndrome Toby C. Trujillo, Pharm.D., BCPS (AQ Cardiology) PRESENTATION

Placebo + ASA*

N = 6303

Clopidogrel + ASA*

N = 6259

• Major bleeding 2.7% 3.7%**

Life threatening bleeding 1 8% 2 2% †

Endpoint

CURE: Bleeding Results

Life-threatening bleeding 1.8% 2.2% †

Non-life-threatening bleeding 0.9% 1.5% ‡

• Minor bleeding 2.4% 5.1% §

* In combination with standard therapy**p = 0.001; †p = NS; ‡p = 0.002; §p < 0.001

Yusuf S et al. N Engl J Med. 2001; 345:494-502.

Composite of MI or cardiovascular deathComposite of MI or cardiovascular deathfrom randomization to end of followfrom randomization to end of follow--upup

PCIPCI--CURE: ResultsCURE: Results

0.150.15

0.100.10

12.6%12.6%

8.8%8.8%

PlaceboPlacebo+ ASA+ ASA

zard

Rat

eza

rd R

ate

31%31%RRRRRR

0 00 0

0.050.05

0.00.000 100100 200200 300300 400400

Days of FollowDays of Follow--upup

pp = = 0.0020.002n = 2658n = 2658

ClopidogrelClopidogrel+ ASA+ ASA

Cu

mu

lati

ve H

azC

um

ula

tive

Haz

Mehta SR et al. Lancet. 2001; 358; 527-33.

dp

oin

t d

po

int

e (%

)e

(%)

No preNo pre--treatment treatment -- PlaceboPlacebo**

1010 Death, MI, or UTVRDeath, MI, or UTVR

18.5% 18.5% RRRRRR

P=0.23P=0.23

99

88

77

66

8.3%8.3%

6.8%6.8%

CREDOEarly Effects of Pre-treatment with

Clopidogrel – 28-Day Results

Co

mb

ined

En

dC

om

bin

ed E

nd

Occ

urr

ence

Occ

urr

ence

Days from RandomizationDays from Randomization

00

55

00 77 1414 2121 2828

*Plus ASA and other standard therapiesUTVR = urgent target vessel revascularization

11

44

33

22

66

Steinhubl SR et al. JAMA. 2002; 288:2411-20.

PrePre--treatment treatment –– ClopidogrelClopidogrel**

13

Page 14: Optimizing Oral Antiplatelet Therapy in Acute Coronary ...Optimizing Oral Antiplatelet Therapy in Acute Coronary Syndrome Toby C. Trujillo, Pharm.D., BCPS (AQ Cardiology) PRESENTATION

Clopidogrel in ACS• NSTE ACS

– CURE (n=12,562)• 300/75 mg vs. placebo within 24 hours

of symptom onset• Significant reductions in death, MI,

and stroke

– PCI-CURE (n=2658)• 31% RRR in CV death or MI

– CREDO (n=2116)

• STEMI– CLARITY (n=3491)

• Patients age 18-75 yr within 12 hours

– Clopidogrel 300/75 mg– Placebo

• Significant reductions in occluded vessel, death, MI by angiography and up to 30 days

– COMMIT (n=45 852)• 300/75 mg vs. placebo 3-24 hours

before PCI, 75 mg vs. placebo after 28 days

• 27% RRR MI/stroke death 1 year• 18.5% RRR in death, MI, UTVR• Benefit only seen if load > 6 hours

before PCI

– ACUITY (n=13,819)• Benefit with bivalirudin monotherapy

contigent on clopidogrel loading pre-PCI

– COMMIT (n=45,852)• Patients within 24 hours of

suspected MI– Clopidogrel 75 mg– Placebo

• 9% reduction in death/MI/stroke over treatment period

Yusuf S et al. N Engl J Med. 2001; 345:494-502; Mehta SR et al. Lancet. 2001; 358:527-33;Steinhubl SR et al. JAMA. 2002; 288:2411-20; Stone GW et al. N Engl J Med. 2006; 355:2203-16; Sabatine MS et al. N Engl J Med. 2005; 352:1179-89; Chen ZM et al. Lancet. 2005; 366:1607-21.

Limitations of Dual Antiplatelet Therapy

ASA plus Clopidogrel• Increased bleeding

• Delayed onset

• Modest level of platelet inhibition

• Variable response– Pharmacogenomics

– Drug interactions

• Irreversible mechanism of action

• Delayed offset

• Residual clinical events

Which of the following statements is true?

a. No clinical benefit is derived with increasing clopidogrel to 150 mg/day

b. Prasugrel reduces risk of CV mortality compared with clopidogrel in treatment of ACS

c. Higher aspirin dose further reduces mortality compared with aspirin 81 mg daily

d. Prasugrel increases risk of bleeding compared with clopidogrel in treatment of ACS

14

Page 15: Optimizing Oral Antiplatelet Therapy in Acute Coronary ...Optimizing Oral Antiplatelet Therapy in Acute Coronary Syndrome Toby C. Trujillo, Pharm.D., BCPS (AQ Cardiology) PRESENTATION

CURRENT Trial: Study DesignACS Patients (UA/NSTEMI 70.8%, STEMI 29.2%)

Planned early (<24 hours) invasive management with intended PCI Ischemic ECG Δ (80.8%) or ↑cardiac biomarker (42%)

Clopidogrel 600Clopidogrel 600--mg load,mg load,150 mg/day x 7 days,150 mg/day x 7 days,

then 75 mg/daythen 75 mg/day

Clopidogrel 300Clopidogrel 300--mg load,mg load,then 75 mg/daythen 75 mg/day

N = 25,087N = 25,087

Efficacy outcomes:Efficacy outcomes: CV death, MI, or stroke at day 30CV death, MI, or stroke at day 30Stent thrombosis at day 30Stent thrombosis at day 30

Safety outcomes:Safety outcomes: Bleeding Bleeding Key subgroup: Key subgroup: PCI vs. no PCIPCI vs. no PCI

Aspirin 300-325 mg/day

Aspirin 75-100 mg/day

Aspirin 300-325 mg/day

Aspirin 75-100 mg/day

PCI 17,232 (70%)PCI 17,232 (70%) No PCI 7855 (30%)No PCI 7855 (30%)

CURRENT-OASIS 7 Investigators et al. N Engl J Med. 2010; 363:930-42.

CURRENT Trial Results: Clopidogrel Double vs. Standard Dose

Standard Double HR 95% CI P Intn P

CV death/MI/stroke

PCI (2N = 17,232) 4.5 3.9 0.85 0.74-0.99 0.0360.016

No PCI (2N = 7855) 4.2 4.9 1.17 0.95-1.44 0.14

Overall (2N = 25,087) 4.4 4.2 0.95 0.84-1.07 0.370

MI

PCI (2N = 17,232) 2.6 2.0 0.78 0.64-0.95 0.0120.025

No PCI (2N = 7855) 1.4 1.7 1.25 0.87-1.79 0.23

Overall (2N = 25,087) 2.2 1.9 0.86 0.73-1.03 0.097

CV Death

PCI (2N = 17,232) 1.9 1.9 0.96 0.77-1.19 0.681.0

No PCI (2N = 7855) 2.8 2.7 0.96 0.74-1.26 0.77

Overall (2N = 25,087) 2.2 2.1 0.96 0.81-1.14 0.628

Stroke

PCI (2N = 17,232) 0.4 0.4 0.88 0.55-1.41 0.590.50

No PCI (2N = 7855) 0.8 0.9 1.11 0.68-1.82 0.67

Overall (2N = 25,087) 0.5 0.5 0.99 0.70-1.39 0.950

CURRENT-OASIS 7 Investigators et al. N Engl J Med. 2010; 363:930-42.

CURRENT Trial Results: Clopidogrel Double vs. Standard Dose

CURRENT-OASIS 7 Investigators et al. N Engl J Med. 2010; 363:930-42.

Enlarged slide on page 21

15

Page 16: Optimizing Oral Antiplatelet Therapy in Acute Coronary ...Optimizing Oral Antiplatelet Therapy in Acute Coronary Syndrome Toby C. Trujillo, Pharm.D., BCPS (AQ Cardiology) PRESENTATION

CURRENT Trial Results: ASA High Dose vs. Low Dose

CURRENT-OASIS 7 Investigators et al. N Engl J Med. 2010; 363:930-42.

P2Y12 Receptor Inhibitors• Clopidogrel

– Prodrug: 2-step cytochrome P-450 conversion• 2C19 patient variation• Drug interactions (PPIs)• Large amount of patient variability in response

– Onset delayed– Irreversible inhibition of the P2Y12 receptor (life of the

platelet)platelet)

• Prasugrel– Prodrug: Single-step cytochrome P-450 conversion

• Multiple enzymes capable of conversion• Lacks patient variability compared to clopidogrel• Provides more potent platelet inhibition compared with clopidogrel

– Onset in about 30 minutes– Irreversible inhibition of the P2Y12 receptor (life of the

platelet)PPIs = proton pump inhibitors

Biotransformation and Mode of ActionClopidogrel, Prasugrel, and Ticagrelor

Schömig A. N Engl J Med. 2009; 361:1108-11.

Enlarged slide on page 22

Enlarged slide on page 22

16

Page 17: Optimizing Oral Antiplatelet Therapy in Acute Coronary ...Optimizing Oral Antiplatelet Therapy in Acute Coronary Syndrome Toby C. Trujillo, Pharm.D., BCPS (AQ Cardiology) PRESENTATION

TRITON-TIMI 38ACS (STEMI or UA/NSTEMI) and planned PCI

PRASUGREL60-mg loading dose

10-mg maintenance dose

CLOPIDOGREL300-mg loading dose

75-mg maintenance dose

N = 13,608

1o endpoint: CV death, MI, stroke2o endpoints: CV death, MI, stroke, rehospitalization due to recurrent ischemia

CV death, MI, UTVRStent thrombosis (ARC definite/probable)

Safety endpoints: TIMI major bleeds, life-threatening bleeds

Median duration of therapy—14.5 months

ARC = Academic Research ConsortiumWiviott SD et al. N Engl J Med. 2007; 357:2001-15.

TRITON-TIMI 38 Trial Results

10

15

HR 0.81(0.73-0.90)p = 0.0004

Prasugrel

Clopidogrel

dp

oin

t (%

)

12.1

9.9

138Events

CV death/MI/Stroke

NNT = 46

0

5

0 30 60 90 180 270 360 450Days

En

d

HR 1.32(1.03-1.68)

p = 0.03

Prasugrel

Clopidogrel1.8

2.4

35EventsTIMI Major and

Non-CABG Bleeds

NNH = 167

NNT = number needed to treatNNH = number needed to harm

120 150 210 240 300 330 390 420

Wiviott SD et al. N Engl J Med. 2007; 357:2001-15.

TRITON-TIMI 38 Results

Wiviott SD et al. N Engl J Med. 2007; 357:2001-15.

Enlarged slide on page 23

17

Page 18: Optimizing Oral Antiplatelet Therapy in Acute Coronary ...Optimizing Oral Antiplatelet Therapy in Acute Coronary Syndrome Toby C. Trujillo, Pharm.D., BCPS (AQ Cardiology) PRESENTATION

TRITONTRITON--TIMI 38 TrialTIMI 38 TrialBleeding OutcomesBleeding Outcomes

NNHNNH

167167

200200

Wiviott SD et al. N Engl J Med. 2007; 357:2001-15.

200200

250250

334334

84841001001010

TRITON-TIMI 38 TrialBleeding Risk Subgroups

≥ 75 yr≥ 75 yr

NoNo

YesYesPriorPriorStroke / TIAStroke / TIA

Risk (%)Risk (%)

+ 37+ 37

-1616

--11

pp = 0.006= 0.006

PostPost--hoc analysishoc analysis

OVERALLOVERALL

≥ 60 kg

< 60 kg< 60 kg

< 75 yr< 75 yr

≥ 75 yr≥ 75 yr

0.5 1 2

AgeAge

WeightWeight

--1616

+3+3

--1414

--1313

Prasugrel BetterPrasugrel Better Clopidogrel BetterClopidogrel BetterHRHR

pp = 0.18= 0.18

pp = 0.36= 0.36

TRITON-TIMI 38 TrialNet Clinical Benefit by Subgroup

Prior stoke or TIA HR 1.54 (95% CI 1.02–2.32); p=0.04

≥ 75 Years of age HR 0.99 (95% CI 0.81–1.21); p=0.92

Prasugrel vs. Clopidogrel by Subgroup

3.8%

g ( ); p

Weight ≤ 60 kg HR 1.03 (95% CI 0.69–1.53); p=0.89

16%

80% of patients in TRITON-TIMI 38 demonstrated a significant reduction in CV death, MI, or stroke

without an increase in bleeding

TIA = transient ischemic attackWiviott SD et al. N Engl J Med. 2007; 357:2001-15.

Enlarged slide on page 23

18

Page 19: Optimizing Oral Antiplatelet Therapy in Acute Coronary ...Optimizing Oral Antiplatelet Therapy in Acute Coronary Syndrome Toby C. Trujillo, Pharm.D., BCPS (AQ Cardiology) PRESENTATION

TRITON-TIMI 38 TrialSubgroups

Diabetes STEMI

Montalescot G. Eur Heart J. 2009; 11(suppl G):G18-G24.

2009 ACCF/AHA PCI GuidelinesDuration of Thienopyridine Therapy

• In patients receiving a stent (BMS or DES) during PCI for ACS (Class I recommendation)– Clopidogrel 75 mg daily for at least 12 months (Class I

recommendation) and up to 15 months (Class I recommendation) or– Prasugrel 10 mg daily for at least 12 months (Class I

recommendation) or at least 15 months (Class IIb recommendation)

• Continuation of clopidogrel or prasugrel beyond 15 months may be considered in patients undergoing DES placement (Class IIb recommendation)

• If the risk of morbidity because of bleeding outweighs the anticipated benefit afforded by thienopyridine therapy, earlier discontinuation should be considered (Class I recommendation)

Kushner FG et al. Circulation. 2009; 120:2271-306.

2009 ACCF/AHA PCI Guidelines

• Conservative strategy (medical therapy without PCI ± stent)

Clopidogrel loading dose 300 mg followed Clopidogrel loading dose 300 mg, followed by 75 mg daily at least one month and ideally up to 1 year

King SB 3rd et al. J Am Coll Cardiol. 2008; 51:172-209.

Enlarged slide on page 24

19

Page 20: Optimizing Oral Antiplatelet Therapy in Acute Coronary ...Optimizing Oral Antiplatelet Therapy in Acute Coronary Syndrome Toby C. Trujillo, Pharm.D., BCPS (AQ Cardiology) PRESENTATION

Evolution of Antiplatelet Therapy in ACS

1 08

ASA

ASA + ClopidogrelASA +

Prasugrel- 22%

- 20%

19%

Reductionin

IschemicEvents

0

Placebo ATTC CURE TRITON-TIMI 38

Single Antiplatelet Rx

Dual Antiplatelet Rx

Higher IPA

- 19%

+ 60% + 38% + 32%

Increasein

Major Bleeds

Placebo ATTC CURE TRITON-TIMI 38

IPA = Inhibition of platelet aggregation

Conclusion

• Dual antiplatelet therapy represents a significant advance in the treatment of patients with ACS

• Despite proven benefits not all patients• Despite proven benefits, not all patients benefit equally with ASA plus clopidogrel

• Emerging evidence suggests additional benefits may be realized with new agents or altered dosing

20

Page 21: Optimizing Oral Antiplatelet Therapy in Acute Coronary ...Optimizing Oral Antiplatelet Therapy in Acute Coronary Syndrome Toby C. Trujillo, Pharm.D., BCPS (AQ Cardiology) PRESENTATION

Existing and Future Antiplatelet Agents

Angiolillo DJ et al. Eur Heart J. 2010; 31:17-28.

CURRENT Trial Results: Clopidogrel Double vs. Standard Dose

CURRENT-OASIS 7 Investigators et al. N Engl J Med. 2010; 363:930-42.

21

Page 22: Optimizing Oral Antiplatelet Therapy in Acute Coronary ...Optimizing Oral Antiplatelet Therapy in Acute Coronary Syndrome Toby C. Trujillo, Pharm.D., BCPS (AQ Cardiology) PRESENTATION

CURRENT Trial Results: ASA High Dose vs. Low Dose

CURRENT-OASIS 7 Investigators et al. N Engl J Med. 2010; 363:930-42.

Biotransformation and Mode of ActionClopidogrel, Prasugrel, and Ticagrelor

Schömig A. N Engl J Med. 2009; 361:1108-11.

22

Page 23: Optimizing Oral Antiplatelet Therapy in Acute Coronary ...Optimizing Oral Antiplatelet Therapy in Acute Coronary Syndrome Toby C. Trujillo, Pharm.D., BCPS (AQ Cardiology) PRESENTATION

TRITON-TIMI 38 Results

Wiviott SD et al. N Engl J Med. 2007; 357:2001-15.

TRITONTRITON--TIMI 38 TrialTIMI 38 TrialBleeding OutcomesBleeding Outcomes

NNHNNH

167167

200200

200200

250250

Wiviott SD et al. N Engl J Med. 2007; 357:2001-15.

334334

84841001001010

23

Page 24: Optimizing Oral Antiplatelet Therapy in Acute Coronary ...Optimizing Oral Antiplatelet Therapy in Acute Coronary Syndrome Toby C. Trujillo, Pharm.D., BCPS (AQ Cardiology) PRESENTATION

TRITON-TIMI 38 TrialSubgroups

Di b t STEMIDiabetes STEMI

Montalescot G. Eur Heart J. 2009; 11(suppl G):G18-G24.

24

Page 25: Optimizing Oral Antiplatelet Therapy in Acute Coronary ...Optimizing Oral Antiplatelet Therapy in Acute Coronary Syndrome Toby C. Trujillo, Pharm.D., BCPS (AQ Cardiology) PRESENTATION

Optimizing Oral Antiplatelet Therapy in Acute Coronary Syndrome

S E L E C T E D R E F E R E N C E S

1. Anderson JL, Adams CD, Antman EM et al. ACC/AHA 2007 guidelines for the management of patients with unstable angina/non-ST-Elevation myocardial infarction: a report of the American College of Cardiology/American Heart Association Task Force on Practice Guidelines (Writing Committee to Revise the 2002 Guidelines for the Management of Patients With Unstable Angina/Non-ST-Elevation Myocardial Infarction) developed in collaboration with the American College of Emergency Physicians, the Society for Cardiovascular Angiography and Interventions, and the Society of Thoracic Surgeons endorsed by the American Association of Cardiovascular and Pulmonary Rehabilitation and the Society for Academic Emergency Medicine. J Am Coll Cardiol. 2007; 50:e1-e157.

2. Angiolillo DJ, Capodanno D, GotoS. Platelet thrombin receptor antagonism and atherothrombosis. Eur Heart J. 2010; 31:17-28.

3. Antithrombotic Trialists’ Collaboration. Collaborative meta-analysis of randomised trials of antiplatelet therapy for prevention of death, myocardial infarction, and stroke in high risk patients. BMJ. 2002; 324:71-86.

4. Chen ZM, Jiang LX, Chen YP et al. Addition of clopidogrel to aspirin in 45,852 patients with acute myocardial infarction: randomised placebo-controlled trial. Lancet. 2005; 366:1607-21.

5. CURRENT-OASIS 7 Investigators, Mehta SR, Bassand JP et al. Dose comparisons of clopidogrel and aspirin in acute coronary syndromes. N Engl J Med. 2010; 363:930-42.

6. ISIS-2 (Second International Study of Infarct Survival) Collaborative Group. Randomised trial of intravenous streptokinase, oral aspirin, both, or neither among 17,187 cases of suspected acute myocardial infarction: ISIS-2. Lancet. 1988; 2:349-60.

7. King SB 3rd, Smith SC Jr, Hirshfeld JW Jr et al. 2007 focused update of the ACC/AHA/SCAI 2005 guideline update for percutaneous coronary intervention: a report of the American College of Cardiology/American Heart Association Task Force on Practice Guidelines. J Am Coll Cardiol. 2008; 51:172-209.

8. Kushner FG, Hand M, Smith SC Jr et al. 2009 focused updates: ACC/AHA guidelines for the management of patients with ST-elevation myocardial infarction (updating the 2004 guideline and 2007 focused update) and ACC/AHA/SCAI guidelines on percutaneous coronary intervention (updating the 2005 guideline and 2007 focused update): a report of the American College of Cardiology Foundation/American Heart Association Task Force on Practice Guidelines. Circulation. 2009; 120:2271-306.

9. Mehta SR, Yusuf S, Peters RJ et al. Effects of pretreatment with clopidogrel and aspirin followed by long-term therapy in patients undergoing percutaneous coronary intervention: the PCI-CURE study. Lancet. 2001; 358:527-33.

10. Montalescot G. Benefits for specific subpopulations in TRITON-TIMI 38. Eur Heart J. 2009; 11(suppl G):G18-G24.

25

Page 26: Optimizing Oral Antiplatelet Therapy in Acute Coronary ...Optimizing Oral Antiplatelet Therapy in Acute Coronary Syndrome Toby C. Trujillo, Pharm.D., BCPS (AQ Cardiology) PRESENTATION

Optimizing Oral Antiplatelet Therapy in Acute Coronary Syndrome

11. RISC Group. Risk of myocardial infarction and death during treatment with low dose

aspirin and intravenous heparin in men with unstable coronary artery disease. The RISC Group. Lancet. 1990; 336:827-30.

12. Roux S, Christeller S, Lüdin E. Effects of aspirin on coronary reocclusion and recurrent ischemia after thrombolysis: a meta-analysis. J Am Coll Cardiol.1992; 19:671-7.

13. Sabatine MS, Cannon CP, Gibson CM et al. Addition of clopidogrel to aspirin and fibrinolytic therapy for myocardial infarction with ST-segment elevation. N Engl J Med. 2005; 352:1179-89.

14. Schömig A. Ticagrelor--is there need for a new player in the antiplatelet-therapy field? N Engl J Med. 2009; 361:1108-11.

15. Steinhubl SR, Berger PB, Mann JT 3rd et al. Early and sustained dual oral antiplatelet therapy following percutaneous coronary intervention: a randomized controlled trial. JAMA. 2002; 288:2411-20.

16. Stone GW, McLaurin BT, Cox DA et al. Bivalirudin for patients with acute coronary syndromes. N Engl J Med. 2006; 355:2203-16.

17. Wiviott SD, Braunwald E, McCabe CH et al. for the TRITON-TIMI 38 Investigators. Prasugrel versus clopidogrel in patients with acute coronary syndromes. N Engl J Med. 2007; 357:2001-15.

18. Yusuf S, Zhao F, Mehta SR et al. Effects of clopidogrel in addition to aspirin in patients with acute coronary syndromes without ST-segment elevation. N Engl J Med. 2001; 345:494-502.

26

Page 27: Optimizing Oral Antiplatelet Therapy in Acute Coronary ...Optimizing Oral Antiplatelet Therapy in Acute Coronary Syndrome Toby C. Trujillo, Pharm.D., BCPS (AQ Cardiology) PRESENTATION

Optimizing Oral Antiplatelet Therapy in Acute Coronary Syndrome

S E L F – A S S E S S M E N T Q U E S T I O N S

1. Which of the following statements is true regarding the use of aspirin or clopidiogrel or both in the treatment of acute coronary syndrome?

a. The combination of aspirin plus clopidogrel does not increase the risk of bleeding

compared with aspirin alone. b. The combination of aspirin plus clopidogrel reduces the risk of recurrent MI

compared with aspirin alone. c. In the CURRENT trial, a daily dose of aspirin 300-325 mg led to a reduction in

the risk of myocardial infarction (MI) compared with lower daily dose of 75-100 mg.

d. In the CURRENT trial, a daily dose of aspirin 300-325 mg led to a reduction in the risk of total mortality compared with lower daily dose of 75-100 mg.

2. All of the following are considered limitations with dual antiplatelet therapy of aspirin

plus clopidogrel EXCEPT

a. Increased bleeding compared with aspirin monotherapy. b. Variable antiplatelet response with clopidogrel. c. Quick offset of action with drug discontinuation. d. Irreversible inhibition of platelet function.

3. The Triton-TIMI 38 study comparing clopidogrel and prasugrel in patients with acute

coronary syndrome and planned percutaneous coronary intervention showed that

a. Patients with a previous history of stroke had an overall net benefit with prasugrel compared with clopidogrel.

b. Patients with a previous history of diabetes had an overall net benefit with prasugrel compared with clopidogrel.

c. Patients aged 75 and greater had an overall net benefit with prasugrel compared with clopidogrel.

d. Patients weighing less than 60 kg had an overall net benefit with prasugrel compared with clopidogrel.

Answers

1. b 2. c 3. b

27

Page 28: Optimizing Oral Antiplatelet Therapy in Acute Coronary ...Optimizing Oral Antiplatelet Therapy in Acute Coronary Syndrome Toby C. Trujillo, Pharm.D., BCPS (AQ Cardiology) PRESENTATION

Optimizing Oral Antiplatelet Therapy in Acute Coronary Syndrome

28

Page 29: Optimizing Oral Antiplatelet Therapy in Acute Coronary ...Optimizing Oral Antiplatelet Therapy in Acute Coronary Syndrome Toby C. Trujillo, Pharm.D., BCPS (AQ Cardiology) PRESENTATION

Optimizing Oral Antiplatelet Therapy in Acute Coronary Syndrome

Julie H. Oestreich, Pharm.D., Ph.D. Assistant Professor of Pharmacy Practice College of Pharmacy University of Nebraska Medical Center Omaha, Nebraska Julie H. Oestreich, Pharm.D., Ph.D., is Assistant Professor in the Department of Pharmacy Practice in the College of Pharmacy at the University of Nebraska Medical Center (UNMC) in Omaha, Nebraska. Dr. Oestreich earned her Doctor of Pharmacy degree from Ohio Northern University in Ada, Ohio, and her Doctor of Philosophy in Pharmaceutical Sciences degree from the University of Kentucky College of Pharmacy in Lexington, Kentucky. She was also awarded a Graduate Certificate in Clinical Research Skills. She completed a post-doctoral fellowship at the Saha Cardiovascular Research Center in the College of Medicine at the University of Kentucky. This fellowship was supported by a Ruth L. Kirschstein National Research Service Award Institutional Training Grant (T32). At the UNMC College of Pharmacy, Dr. Oestreich assists in the cardiology therapeutics module and coordinates an elective in pharmacogenomics. Her research specialty is platelet function testing and evaluating the effectiveness of antiplatelet agents. Her dissertation entitled, “Variability in ADP-mediated Platelet Response,” focused on platelet function in healthy volunteers, and she continues to study the variability of P2Y12 and other platelet receptor pathways in humans and in animal models. Dr. Oestreich has received competitive fellowship support from the American Heart Association, Pharmaceutical Research and Manufacturers of America Foundation, American Foundation for Pharmaceutical Education, and Sigma Xi Scientific Research Society. Dr. Oestreich is an active member of the American College of Clinical Pharmacy and currently serves as chair of its Pharmacokinetics and Pharmacodynamics Practice and Research Network. She is also an active member of the American Pharmacists Association (APhA) and will serve as chair-elect of the Clinical Sciences Section of the APhA Academy of Pharmaceutical Research and Science starting in 2011. She has authored several articles and national abstracts in the area of platelet function and antiplatelet effectiveness.

29

Page 30: Optimizing Oral Antiplatelet Therapy in Acute Coronary ...Optimizing Oral Antiplatelet Therapy in Acute Coronary Syndrome Toby C. Trujillo, Pharm.D., BCPS (AQ Cardiology) PRESENTATION

Optimizing Oral Antiplatelet Therapy in Acute Coronary Syndrome

Julie H. Oestreich, Pharm.D., Ph.D. P R E S E N T A T I O N

Clinical Challenges in Oral Antiplatelet Therapy O V E R V I E W

Increasing evidence supports relevant variability in clopidogrel effectiveness as measured by platelet function tests. Several clinical studies in patients treated with aspirin and clopidogrel have associated high platelet reactivity with adverse cardiovascular events. The potential usefulness of platelet function testing is further magnified by the availability of more potent P2Y12 receptor antagonists, including prasugrel and potentially ticagrelor. Thus far, platelet tests have been more successful at identifying patients with lower risk than predicting which patients will experience adverse outcomes and, therefore, have not been widely used. Other unique challenges of clopidogrel therapy include recent FDA labeling changes regarding the drug interaction with omeprazole and reduced effectiveness of clopidogrel in CYP2C19 loss-of-function allele carriers. The strong FDA labeling change for omeprazole and other potent CYP2C19 inhibitors is based on consistent pharmacokinetic and pharmacodynamic effects, but the clinical relevance of this interaction is still being debated. Similarly, patients who have one or two loss-of-function alleles for the CYP2C19 gene demonstrate decreased active metabolite formation and impaired platelet inhibition with clopidogrel. The impact of CYP2C19 carrier status on cardiovascular events has varied in retrospective analyses of large-scale clinical trials, although the combined evidence suggests a small, yet significant effect. At the present time, routine genetic or platelet function testing is not recommended; nonetheless, high platelet reactivity and CYP2C19 metabolizer status, if known, should be taken into consideration—along with other clinical factors known to influence clopidogrel effectiveness and bleeding risk—when assessing appropriate antiplatelet therapy.

30

Page 31: Optimizing Oral Antiplatelet Therapy in Acute Coronary ...Optimizing Oral Antiplatelet Therapy in Acute Coronary Syndrome Toby C. Trujillo, Pharm.D., BCPS (AQ Cardiology) PRESENTATION

Clinical Challenges in Oral Antiplatelet Therapy

Julie H. Oestreich, Pharm.D., Ph.D.Assistant Professor of Pharmacy Practice

College of PharmacyUniversity of Nebraska Medical Center

Omaha, Nebraska

Which of the following is TRUE regarding clopidogrel variability?a. Platelet reactivity on clopidogrel is a dichotomous

variable allowing for highly sensitive, specific cut-off values for predicting CV events

b. Platelet function tests for clopidogrel have low negative predictive values for CV eventspredictive values for CV events

c. Several prospective clinical trials attributed CV events on clopidogrel to PPI use

d. Patients with high platelet reactivity on clopidogrel have increased risk for CV events

CLOPIDOGREL VARIABILITYClinical Challenge #1:

CLOPIDOGREL VARIABILITY

31

Page 32: Optimizing Oral Antiplatelet Therapy in Acute Coronary ...Optimizing Oral Antiplatelet Therapy in Acute Coronary Syndrome Toby C. Trujillo, Pharm.D., BCPS (AQ Cardiology) PRESENTATION

Biotransformation and Mode of Action:Clopidogrel, Prasugrel, and Ticagrelor

Schömig Aet al. N Engl J Med. 2009; 361:1108-11.

Distribution of Clopidogrel Response in 544 Volunteers

80

96

112

A Normal Distribution: Consistent with a Poly-Genetic and Poly-Environmental Influence

Change in Aggregation to 5 μmol/L ADP

Number of patients

0

16

32

48

64

80

<= -20 [-10,0] [11,20] [31,40] [51,60] [71,80] [91,100]

Serebruany VL et al. J Am Coll Cardiol. 2005; 45:246-51.

Terminology

• Clopidogrel ‘resistance’

• Clopidogrel nonresponsiveness

• High on-treatment platelet reactivity

Context

• Pharmacokinetic formation of active metabolite

• Pharmacodynamic platelet inhibition

• Cardiovascular events on compliant aspirin and clopidogrel treatment

Enlarged slide on page 22

32

Page 33: Optimizing Oral Antiplatelet Therapy in Acute Coronary ...Optimizing Oral Antiplatelet Therapy in Acute Coronary Syndrome Toby C. Trujillo, Pharm.D., BCPS (AQ Cardiology) PRESENTATION

Bleeding Bleeding timetime LTALTA VerifyNowVerifyNow TEGTEG

Flow Flow cytometrycytometry

PlateletworksPlateletworks

VASP VASP phosphorylationphosphorylation

MultiplateMultiplateanalyzeranalyzer

P-VASP VASPP-VASP VASP

Platelet Activation

OthersOthers

Impedance Impedance aggregometryaggregometry

Which Assay Is Best?

Assay LTA VASP Multiplate VerifyNow

Advantages VersatileSmall whole

blood volumePhysiologic environment

Standardized

Long history-S iti t

MinimalU f i dl

Long historycorrelated

with outcomes

Sensitive to P2Y12 effect

Minimalsample

manipulation

User friendly and fast

DisadvantagesNot

standardizedExpensive

Lessexperience

Expensive

Specializeduser

Specializeduser

Oestreich JH et al. Thromb Haemost. 2009; 101:217-9.

Correlation of Clopidogrel Responsiveness with Clinical Outcomes

• One of first trials to correlate lower antiplatelet effects of clopidogrel to clinical outcomes– 60 patients with STEMI with PCI + stent

– Clopidogrel 300 mg LD, then 75 mg daily

– Platelet activity assessed at PCI and daily x 5 days

• Patients separated into quartiles based on ADP-induced platelet aggregation

• Major adverse cardiac events evaluated at 6 months

Matetzky S et al. Circulation. 2004; 109:3171-5.

33

Page 34: Optimizing Oral Antiplatelet Therapy in Acute Coronary ...Optimizing Oral Antiplatelet Therapy in Acute Coronary Syndrome Toby C. Trujillo, Pharm.D., BCPS (AQ Cardiology) PRESENTATION

Correlation of Clopidogrel Responsiveness with Clinical Outcomes

30%

40% 40% p=0.007

6-Month Recurrent CV Events (%)

0%

10%

20%

1st 2nd 3rd 4th

6.7% 0% 0%

Matetzky S et al. Circulation. 2004; 109:3171-5.

n=15 n=15 n=15 n=15

High On-treatment Platelet Reactivity: PREPARE POST-STENTING Trial

• Patients undergoing elective PCI + stent (n=192) receiving clopidogrel– 600 mg LD (n=60); 300 mg LD (n=75)

– Already on 75 mg daily (n=57)y g y ( )

• Platelet tests (pretreatment and ~24 hours post-PCI)

– LTA with ADP 20 μmol/L– Thrombelastograph (TEG)

• Results at 6 months– 20% of patient with MACE (n=38)

– 80% of patients event free (n=154)

Gurbel PA et al. J Am Coll Cardiol. 2005; 46:1820-6.

PREPARE POST-STENTING Trial

Patients withPatients with Patients withoutPatients withoutIschemic EventsIschemic Events Ischemic Events Ischemic Events pp--valuevalue

LTA-ADP 20 pre-treatment (%) 71 ± 9 73 ± 12 NSLTA-ADP 20 post-treatment (%) 63 ± 12 56 ± 16 0.02

TEG MA pre-treatment (mm) 72 ± 7 67 ± 8 <0.001TEG MA post-treatment (mm) 74 ± 5 65 ± 5 <0.001

Gurbel PA et al. J Am Coll Cardiol. 2005; 46:1820-6.

Lowest LTA post-treatment quartile (<50% platelet aggregation) – MACE 10%Highest LTA post-treatment quartile (>67% platelet aggregation) – MACE 32%

p = 0.02

MA = maximal absorption (clot strength)

34

Page 35: Optimizing Oral Antiplatelet Therapy in Acute Coronary ...Optimizing Oral Antiplatelet Therapy in Acute Coronary Syndrome Toby C. Trujillo, Pharm.D., BCPS (AQ Cardiology) PRESENTATION

Other Studies

• At least 28 studies link on-treatment platelet reactivity and clopidogrel responsiveness to clinical endpoints in PCI patients– n = 46–1608 patients– Multiple assaysp y

• Clinical outcomes– Stent thrombosis– Major adverse cardiac events (death, MI, TVR)– Cardiac death

Bonello L et al. J Am Coll Cardiol. 2010; 56:919-33.

ROC Curve Analyses of Cut-off ValuesStudy Cut-off Endpoint AUC OR

Price et al. > 235 PRU 6-mo post-PCI CVD/MI/ST 0.71

Gurbel et al. ADP 5 > 46% 2-yr post-PCI MACE 0.77 3.9(2008) ADP 20 > 59% 0.78 3.8

Blindt et al. > 48% PRI 6-mo ST 0.79 1.16

Frere et al. ADP 10 > 70% 1-mo post-PCI MACE + stroke 0.74> 53% PRI 0.73

Bonello et al. > 50% PRI 6-mo post-PCI MACE 0.55(2007)

Marcucci et al. ≥ 240 PRU 1-yr CV death + MI 0.66 2.38-2.75

Sibbing et al. > 468 AU/min 30-day ST 0.78 12.0

Cuisset et al. ADP 10 > 67% 1-mo ST 0.69 5.8

Breet et al. Multiple assays 1-yr death, MI, ST, and stroke 0.61-0.63 2.1-2.5

Laboratory Definitions of HighOn-treatment Platelet Reactivity

• PRI > 50% by VASP phosphorylation

• > 235 – 240 PRU (P2Y12 reaction units) by VerifyNow P2Y12 assay

• > 46% maximal ADP 5 μmol/L-induced aggregation by light transmission aggregometry

• > 468 arbitrary aggregation units/min in response to ADP by the Multiplate analyzer

Bonello L et al. J Am Coll Cardiol. 2010; 56:919-33.

35

Page 36: Optimizing Oral Antiplatelet Therapy in Acute Coronary ...Optimizing Oral Antiplatelet Therapy in Acute Coronary Syndrome Toby C. Trujillo, Pharm.D., BCPS (AQ Cardiology) PRESENTATION

Platelet Function Guided TherapyNon-emergent PCI: ACS and stable angina (n = 1122)

Loading doses (LD):ASA 250 mg, clopidogrel 600 mg VASP ≥ 50%

Randomization(n=429)

CONTROL (n =215) VASP-guided LD (n =214)

Up to 3 additional 600-mg LD every 24 hours untilVASP < 50% before PCI

Maintenance doses: ASA 160 mg, clopidogrel 75 mg

1° endpoint: Definite stent thrombosis (ARC definition)

2° endpoints: MACE including CV death, MI, and UTVR TIMI major and minor bleeding at 30 days

Bonello L et al. Am J Cardiol. 2009; 103:5-10.

Platelet Function Guided Therapy

VASP after first LD 66 ± 11 67 ± 10

VASP after sensitization 37 ± 12 p < 0.01

17 patients (8%)

Bonello L et al. Am J Cardiol. 2009;

103:5-10.

Pla

tele

t re

activ

ity(%

)

VASP 1 VASP 2 VASP 3 VASP 4n = 215 n = 215 n= 83 n = 40

Early Definite Stent Thrombosis and Other MACE during 1 Month Follow-up

Control Control VASPVASP--guided guided Group Group GroupGroup

Endpoint n,(%)Endpoint n,(%) (n=214)(n=214) (n=215)(n=215) pp--valuevalue

Acute stent thrombosisAcute stent thrombosis 2 2 (0.9)(0.9) 00 0.250.25

SubSub--acute stent thrombosisacute stent thrombosis 8 (3.7)8 (3.7) 1 (0.5)1 (0.5) 0.020.02

Early DSTEarly DST 10 (4.7)10 (4.7) 1 1 (0.5)(0.5) 0.010.01

Cardiovascular Cardiovascular deathdeath 4 (1.8)4 (1.8) 00 0.060.06

Myocardial infarctionMyocardial infarction 10 (4.8)10 (4.8) 1 1 (0.5)(0.5) 0.010.01

Urgent revascularizationUrgent revascularization 5 5 (2.3)(2.3) 00 0.060.06

All MACEAll MACE 19 (8.9)19 (8.9) 1 1 (0.5)(0.5) <0.001<0.001

Bonello L et al. Am J Cardiol. 2009; 103:5-10.

36

Page 37: Optimizing Oral Antiplatelet Therapy in Acute Coronary ...Optimizing Oral Antiplatelet Therapy in Acute Coronary Syndrome Toby C. Trujillo, Pharm.D., BCPS (AQ Cardiology) PRESENTATION

The Sweet Spot:Is There a Therapeutic Window for

P2Y12 Inhibition?n = 2,533

Incidence of bleeding (%)

Sibbing D et al. J Am Coll Cardiol. 2010; 56:317-8.

Platelet inhibition determined by Multiplate analyzer.

Incidence of ST (%)

Take-home Message:Clopidogrel Nonresponsiveness and

On-treatment Platelet Reactivity

• Clopidogrel variability as measured by platelet function tests correlates with clinical outcomes in patients undergoing PCIg g– Good negative predictive value– Poor positive predictive value

• Using platelet tests for individualized dosing is advancing

– VerifyNow is most convenient assay

• Platelet function test that best predicts clinical outcomes is not known

Clinical Challenge #2:

CLOPIDOGREL-PPI DRUG INTERACTION

37

vyarborough
Rectangle
vyarborough
Rectangle
vyarborough
Rectangle
vyarborough
Rectangle
Page 38: Optimizing Oral Antiplatelet Therapy in Acute Coronary ...Optimizing Oral Antiplatelet Therapy in Acute Coronary Syndrome Toby C. Trujillo, Pharm.D., BCPS (AQ Cardiology) PRESENTATION

Biotransformation of Clopidogrel

Bonello L et al. J Am Coll Cardiol. 2010; 56:919-33.

OCLA Study• Patients undergoing

elective PCI with stenting (n=140)– ASA plus clopidogrel 300

mg LD, 75 mg qd

– Randomized• Placebo x 7 days

• Omeprazole 20 mg qd x 7 days

– VASP at baseline• 83.2% vs. 83.9%; p=NS

– VASP at day 7• Placebo 39.8%

• Omeprazole 51.4%

• p<0.0001

GilardGilard M M et al. et al. J J Am Coll Am Coll CardiolCardiol. . 20082008; 51:256; 51:256--6060..

Clopidogrel andProton Pump Inhibitor Interaction

TRITON-TIMI 38 Trial(n=13,608)

33% on PPI

Clopidogrel:HR 0.94 (0.80 – 1.11)

O’Donoghue O’Donoghue ML ML et al. et al. Lancet. Lancet. 20092009; 374:989; 374:989--9797..

PPI use did not impact- Primary endpoint- MI- Stent thrombosis

Prasugrel:HR 0.99 (0.83 – 1.19)

CV Death, MI, or Stroke

38

Page 39: Optimizing Oral Antiplatelet Therapy in Acute Coronary ...Optimizing Oral Antiplatelet Therapy in Acute Coronary Syndrome Toby C. Trujillo, Pharm.D., BCPS (AQ Cardiology) PRESENTATION

Clopidogrel-PPI Meta-analysis

Odds ratios for MACE according to PPI use (n = 46,037)

Hulot JS et al. J Am Coll Cardiol. 2010; 56:134-43.

PPIs Associated with CV Risk Independent of Clopidogrel Use

Danish cohort study: hazard ratios for MACE according to PPI and clopidogrel use (n = 24,704 on clopidogrel)

Charlot M et al. Ann Intern Med. 2010; 153:378-86.

COGENT Trial Methods

• Phase 3 efficacy and safety study of CGT-2168, a fixed-dose combination of clopidogrel (75 mg) and omeprazole (20 mg) compared with clopidogrel

• Composite endpoint of CV death, non-fatal MI, CABG or PCI or ischemic strokeCABG or PCI, or ischemic stroke

• Target sample size increased from 3200 to ~5000 patients with an accrual period of 1 year and maximum follow up of 2 years

• Study ended at 3761 patients when the sponsor declared bankruptcy

Bhatt DL et al. N Engl J Med. 2010 Oct 6 [Epub ahead of print].

Enlarged slide on page 48

Enlarged slide on page 48

39

Page 40: Optimizing Oral Antiplatelet Therapy in Acute Coronary ...Optimizing Oral Antiplatelet Therapy in Acute Coronary Syndrome Toby C. Trujillo, Pharm.D., BCPS (AQ Cardiology) PRESENTATION

COGENT Trial ResultsEvent rates at 180 days were 5.7% in placebo group and

4.9% in omeprazole group, HR = 0.99 (0.68-1.44), p = 0.96

Bhatt DL et al. N Engl J Med. 2010 Oct 6 [Epub ahead of print].

n = 3761

FDA Advisory and Plavix® Labeling Change Reminder: October 27, 2010

• FDA continues to warn against the concomitant use of clopidogrel and omeprazole– Separating the dose of clopidogrel and omeprazole in

time will not reduce this drug interactiontime will not reduce this drug interaction

• Recommendation applies only to omeprazole and not to all PPIs

• Pantoprazole may be an alternative PPI for consideration

U.S. Food and Drug Administration. (URL in reference list).

Take-home Message:Clopidogrel Drug Interactions

• PK/PD evidence supports an interaction between clopidogrel and PPIs– However, significant effects with platelet function

tests have not always matched clinical outcomesE id f li i l l i fli ti d t• Evidence for clinical relevance is conflicting and not yet convincing

– Retrospective studies limited by general increased cardiovascular risk

– COGENT study used novel formulation

• Regardless of scientific evidence, FDA label leaves little room for alternative interpretations

40

Page 41: Optimizing Oral Antiplatelet Therapy in Acute Coronary ...Optimizing Oral Antiplatelet Therapy in Acute Coronary Syndrome Toby C. Trujillo, Pharm.D., BCPS (AQ Cardiology) PRESENTATION

Clinical Challenge #3:

CLOPIDOGREL AND CYP2C19

CYP2C19 Alleles and Associated Phenotypes

CYP2C19 alleles Phenotype

One or two gain-of-function alleles (*1/*17 or *17/*17)

Ultrarapid metabolizer (UM)( 1/ 17 or 17/ 17)

No variant alleles (*1/*1) Extensive metabolizer (EM)

One loss-of-function allele (*2, *3, others)

Intermediate metabolizer (IM)

Two loss-of-function alleles (*2, *3, others)

Poor metabolizer (PM)

Mega JL et al. N Engl J Med. 2009; 360:354-62.

CYP2C19 Genotype Frequency

European African Chinese ofDescent Descent Asian Origin(n=1356) (n=966) (n=573)

Extensive metabolism:Extensive metabolism:CYP2C19 *1/*1 74% 66% 38%

Intermediate metabolism:CYP2C19 *1/*2 or *1/*3 26% 29% 50%

Poor metabolism:CYP2C19 *2/*2, *2/*3, or *3/*3 2% 4% 14%

Xie HG et al. Annu Rev Pharmacol Toxicol. 2001; 41:815-50.

41

Page 42: Optimizing Oral Antiplatelet Therapy in Acute Coronary ...Optimizing Oral Antiplatelet Therapy in Acute Coronary Syndrome Toby C. Trujillo, Pharm.D., BCPS (AQ Cardiology) PRESENTATION

Clopidogrel and CYP2C19:Pharmacokinetic and Pharmacodynamic Effects

Healthy carriers of one or more CYP2C19 reduced-function alleles had 32% relative reduction in active metabolite

exposure and a 9% absolute reduction in MPA

Clopidogrel 75 mg Clopidogrel 75 mg

Clopidogrel active metabolite exposure

Antiplatelet response by LTA

Mega JL et al. N Engl J Med. 2009; 360:354-62.

AU

C0-t

ΔM

PA

Clopidogrel and CYP2C19 Clinical Outcomes: TRITON TIMI-38

12.1%12%

15%

Carriers

Noncarriers

p = 0.01

th E

ven

t

n = 1477

2.6%

8.0%

0.8%

0%

3%

6%

9%

D/MI/Stroke Stent Thrombosis

Mega JL et al. N Engl J Med. 2009; 360:354-62.

p = 0.02

% P

atie

nts

wit

Clopidogrel and CYP2C19 Clinical Outcomes: Meta-analysis

Odds ratios for MACE according to CYP2C19 (n = 11,959)

Hulot JS et al. J Am Coll Cardiol. 2010; 56:134-43.

42

Page 43: Optimizing Oral Antiplatelet Therapy in Acute Coronary ...Optimizing Oral Antiplatelet Therapy in Acute Coronary Syndrome Toby C. Trujillo, Pharm.D., BCPS (AQ Cardiology) PRESENTATION

Clopidogrel and CYP2C19 Clinical Outcomes: Stent Thrombosis Meta-Analysis

n = 4905

Hulot JS et al. J Am Coll Cardiol. 2010; 56:134-43.

Clopidogrel and CYP2C19 Clinical Outcomes: PLATO

12%

15%

11.2%10 0%

p=0.46Analysis from the PLATO trial

0%

3%

6%

9%

CV Death/MI/Stroke Stent Thrombosis

2.3%

10.0%

1.5%

Carrier

Noncarriern = 9842

Wallentin L et al. Lancet. 2010; 376:1320-8.

Clopidogrel and CYP2C19 Clinical Outcomes: CURE Trial

8 0%

9.5% 10%

8%

10%

12%Carriers

Noncarriersp = NS p = 0.02

(n = 5059)

Str

oke

) 8.0%

7.7%

0%

2%

4%

6%

LOF (*2,*3) GOF (*17)

Paré G et al. N Engl J Med. 2010; 363:1704-14.

CV

Dea

th, M

I, o

r S

at 1

-yea

r (%

)

43

vyarborough
Rectangle
Page 44: Optimizing Oral Antiplatelet Therapy in Acute Coronary ...Optimizing Oral Antiplatelet Therapy in Acute Coronary Syndrome Toby C. Trujillo, Pharm.D., BCPS (AQ Cardiology) PRESENTATION

Usefulness of Genetic Testing for Predicting Cardiac Outcomes

• CYP2C19 is a consistent predictor of clopidogrel PK/PD and is significantly associated with clinical outcomes in some trials

• No prospective studies have evaluated genetic testing and the effectiveness of subsequent treatment changes

• Positive predictive value is estimated at 12-20%

Changes to the Plavix®

(Clopidogrel) Product Label

Plavix prescribing information. 2010 Aug (URL in ref list).

ACCF/AHA:Approaches to the FDA Boxed Warning

• ‘It neither mandates, requires, nor recommends genetic testing, thereby allowing for flexibility in clinical decisions.’

• ‘The evidence base is insufficient to recommend either routine genetic or platelet function testing at the present time.’

Holmes DR Jr et al. J Am Coll Cardiol. 2010; 56:321-41.Society for Cardiovascular Angiography and Interventions et al. Circulation. 2010; 122:537-57.

44

Page 45: Optimizing Oral Antiplatelet Therapy in Acute Coronary ...Optimizing Oral Antiplatelet Therapy in Acute Coronary Syndrome Toby C. Trujillo, Pharm.D., BCPS (AQ Cardiology) PRESENTATION

Take-home Message:Clopidogrel and CYP2C19

• PK/PD and clinical evidence supports a significant effect of CYP2C19 metabolizer status on clopidogrel effectiveness– However, genetic testing demonstrates low

positive predictive valuepositive predictive value– GWAS attributes 12% of variability to CYP2C19

• Key cardiology groups do not recommend routine genetic testing for clopidogrel at this time

– If genetic information is known, it should be considered in context with other risk factors

• We still do not know how to best optimize clopidogrel therapy and improve clinical outcomes

Amish Pharmacogenomics of AntiPlatelet Intervention (PAPI) Study

Genome-wide association study (n = 429)

Shuldiner AR et al. JAMA. 2009; 302:849-57.

Attributes 12% of clopidogrel variability to CYP2C19

Genome-Wide Association Study: PAPI Study

(n = 429)

Shuldiner AR et al. JAMA. 2009; 302:849-57.

45

Page 46: Optimizing Oral Antiplatelet Therapy in Acute Coronary ...Optimizing Oral Antiplatelet Therapy in Acute Coronary Syndrome Toby C. Trujillo, Pharm.D., BCPS (AQ Cardiology) PRESENTATION

Suboptimal Clopidogrel Response

Genetic factors- Polymorphisms of CYP- Polymorphisms of GPIa- Polymorphisms of P2Y12- Polymorphisms of GP IIIa

Cellular factors- Accelerated platelet turnover- Reduced CYP3A metabolic activity- Increased ADP exposure- Up-regulation of the P2Y12 pathway- Up-regulation of the P2Y1 pathway- Up-regulation of P2Y independent

pathways

Clinical factors- Failure to prescribe- Poor compliance- Under dosing- Poor absorption- Drug/drug interactions- Acute coronary syndrome- Diabetes mellitus- Elevated body mass index

AngiolilloAngiolillo DJ DJ et al. et al. J J Am Coll Am Coll CardiolCardiol. . 20072007; 49:1505; 49:1505--1616..

What information is necessary for appropriate clopidogrel prescribing?

Name: Ima Clotter

BMI: 22 Gender: F Age: 62

Platelet reactivity: ____

CYP2C19 t b li t t

Inflammatory index: ____

Coagulation efficiency: ____

CYP2C19 metabolizer status: ____

Calculated target level of P2Y12 receptor inhibition: ____

Predicted dose: ____

P2Y12 antagonist: ____

Clopidogrel:Current Recommendations

Testing Result Recommendation

Indication for clopidogrelNo routine platelet function or genetic

testing is recommended

Low or normal on treatment plateletLow or normal on-treatment platelet reactivity

Follow population-based guidelines

High on-treatment platelet reactivity Consider alternative strategy

CYP2C19*1/*1 Follow population-based guidelines

CYP2C19 loss-of-function polymorphism

Consider alternative agent

*Consider other factors, including age, BMI, DM, and bleeding risk

46

Page 47: Optimizing Oral Antiplatelet Therapy in Acute Coronary ...Optimizing Oral Antiplatelet Therapy in Acute Coronary Syndrome Toby C. Trujillo, Pharm.D., BCPS (AQ Cardiology) PRESENTATION

TC is a 58-year-old man recently started on clopidogrel 75 mg daily. CYP2C19 genotype is CYP2C19*1/*2. VerifyNow test is slightly elevated (PRU = 245). What is the recommended treatment strategy for TC?

a. Continue clopidogrel 75 mg daily

b. Increase clopidogrel maintenance dose to 150 mg daily

c. Start prasugrelc. Start prasugrel

d. Start ticagrelor

47

Page 48: Optimizing Oral Antiplatelet Therapy in Acute Coronary ...Optimizing Oral Antiplatelet Therapy in Acute Coronary Syndrome Toby C. Trujillo, Pharm.D., BCPS (AQ Cardiology) PRESENTATION

Clopidogrel-PPI Meta-analysis

Odds ratios for MACE according to PPI use (n = 46,037)

Hulot JS et al. J Am Coll Cardiol. 2010; 56:134-43.

PPIs Associated with CV Risk Independent of Clopidogrel Use

Danish cohort study: hazard ratios for MACE according to PPI and clopidogrel use (n = 24,704 on clopidogrel)p g ( p g )

Charlot M et al. Ann Intern Med. 2010; 153:378-86.

48

Page 49: Optimizing Oral Antiplatelet Therapy in Acute Coronary ...Optimizing Oral Antiplatelet Therapy in Acute Coronary Syndrome Toby C. Trujillo, Pharm.D., BCPS (AQ Cardiology) PRESENTATION

Optimizing Oral Antiplatelet Therapy in Acute Coronary Syndrome

S E L E C T E D R E F E R E N C E S

1. Abraham NS, Hlatky MA, Antman EM et al. ACCF/ACG/AHA 2010 expert consensus document on the concomitant use of proton pump inhibitors and thienopyridines: a focused update of the ACCF/ACG/AHA 2008 expert consensus document on reducing the gastrointstinal risks of antiplatelet therapy and NSAID use. Circulation. 2010 Nov 8 [Epub ahead of print].

2. Angiolillo DJ, Fernandez-Ortiz A, Bernardo E et al. Variability in individual responsiveness to clopidogrel: clinical implications, management, and future perspectives. J Am Coll Cardiol. 2007; 49:1505-16.

3. Bhatt DL, Cryer BL, Contant CF et al. Clopidogrel with or without omeprazole in coronary artery disease. N Engl J Med. 2010 Oct 6 [Epub ahead of print].

4. Blindt R, Stellbrink K, de Taeye A et al. The significance of vasodilator-stimulated phosphoprotein for risk stratification of stent thrombosis. Thromb Haemost. 2007; 98:1329-34.

5. Bonello L, Camoin-Jau L, Armero S et al. Tailored clopidogrel loading dose according to platelet reactivity monitoring to prevent acute and subacute stent thrombosis. Am J Cardiol. 2009; 103:5-10.

6. Bonello L, Paganelli F, Arpin-Bornet M et al. Vasodilator-stimulated phosphoprotein phosphorylation analysis prior to percutaneous coronary intervention for exclusion of postprocedural major adverse cardiovascular events. J Thromb Haemost. 2007; 5:1630-6.

7. Bonello L, Tantry US, Marcucci R et al. Consensus and future directions on the definition of high on-treatment platelet reactivity to adenosine diphosphate. J Am Coll Cardiol. 2010; 56:919-33.

8. Breet NJ, Van Werkum JW, Bouman HJ et al. Comparison of platelet function tests in predicting clinical outcome in patients undergoing coronary stent implantation. JAMA. 2010; 303:754-62.

9. Charlot M, Ahlehoff O, Norgaard ML et al. Proton-pump inhibitors are associated with increased cardiovascular risk independent of clopidogrel use: a nationwide cohort study. Ann Intern Med. 2010; 153:378-86.

10. Cuisset T, Frere C, Quilici J et al. Predictive values of post-treatment adenosine diphosphate-induced aggregation and vasodilator-stimulated phosphoprotein index for stent thrombosis after acute coronary syndrome in clopidogrel-treated patients. Am J Cardiol. 2009; 104:1078-82.

11. Frere C, Cuisset T, Quilici J et al. ADP-induced platelet aggregation and platelet reactivity index VASP are good predictive markers for clinical outcomes in non-ST elevation acute coronary syndrome. Thromb Haemost. 2007; 98:838-43.

12. Gilard M, Arnaud B, Cornily JC et al. Influence of omeprazole on the antiplatelet action of clopidogrel associated with aspirin: the randomized, double-blind OCLA (Omeprazole CLopidogrel Aspirin) study. J Am Coll Cardiol. 2008; 51:256-60.

49

Page 50: Optimizing Oral Antiplatelet Therapy in Acute Coronary ...Optimizing Oral Antiplatelet Therapy in Acute Coronary Syndrome Toby C. Trujillo, Pharm.D., BCPS (AQ Cardiology) PRESENTATION

Optimizing Oral Antiplatelet Therapy in Acute Coronary Syndrome

13. Gurbel PA, Antonino MJ, Bliden KP et al. Platelet reactivity to adenosine

diphosphate and long-term ischemic event occurrence following percutaneous coronary intervention: a potential antiplatelet therapeutic target. Platelets. 2008; 19:595-604.

14. Gurbel PA, Bliden KP, Guver K et al. Platelet reactivity in patients and recurrent events post-stenting: results of the PREPARE POST-STENTING Study. J Am Coll Cardiol. 2005; 46:1820-6.

15. Gurbel PA, Tantry US, Shuldiner AR et al. Genotyping: one piece of the puzzle to personalize antiplatelet therapy. J Am Coll Cardiol. 2010; 56:112-6.

16. Hochholzer W, Trenk D, Fromm MF et al. Impact of cytochrome P450 2C19 loss-of-function polymorphism and of major demographic characteristics on residual platelet function after loading and maintenance treatment with clopidogrel in patients undergoing elective coronary stent placement. J Am Coll Cardiol. 2010; 55:2427-34.

17. Holmes DR Jr, Dehmer GJ, Kaul S et al. ACCF/AHA clopidogrel clinical alert: approaches to the FDA "boxed warning": a report of the American College of Cardiology Foundation Task Force on clinical expert consensus documents and the American Heart Association endorsed by the Society for Cardiovascular Angiography and Interventions and the Society of Thoracic Surgeons. J Am Coll Cardiol. 2010; 56:321-41.

18. Hulot JS, Collet JP, Silvain J et al. Cardiovascular risk in clopidogrel-treated patients according to cytochrome P450 2C19*2 loss-of-function allele or proton pump inhibitor coadministration: a systematic meta-analysis. J Am Coll Cardiol. 2010; 56:134-43.

19. Jeong YH, Kim IS, Park Y et al. Carriage of cytochrome 2C19 polymorphism is associated with risk of high post-treatment platelet reactivity on high maintenance-dose clopidogrel of 150 mg/day: results of the ACCEL-DOUBLE (Accelerated Platelet Inhibition by a Double Dose of Clopidogrel According to Gene Polymorphism) study. JACC Cardiovasc Interv. 2010; 3:731-41.

20. Marcucci R, Gori AM, Paniccia R et al. Cardiovascular death and nonfatal myocardial infarction in acute coronary syndrome patients receiving coronary stenting are predicted by residual platelet reactivity to ADP detected by a point-of-care assay: a 12-month follow-up. Circulation. 2009; 119:237-342.

21. Matetzky S, Shenkman B, Guetta V et al. Clopidogrel resistance is associated with increased risk of recurrent atherothrombotic events in patients with acute myocardial infarction. Circulation. 2004; 109:3171-5.

22. Mega JL, Close SL, Wiviott SD et al. Cytochrome p-450 polymorphisms and response to clopidogrel. N Engl J Med. 2009; 360:354-62.

23. O’Donoghue ML, Braunwald E, Antman EM et al. Pharmacodynamic effect and clinical efficacy of clopidogrel and prasugrel with or without a proton-pump inhibitor: an analysis of two randomised trials. Lancet. 2009; 374:989-97.

50

Page 51: Optimizing Oral Antiplatelet Therapy in Acute Coronary ...Optimizing Oral Antiplatelet Therapy in Acute Coronary Syndrome Toby C. Trujillo, Pharm.D., BCPS (AQ Cardiology) PRESENTATION

Optimizing Oral Antiplatelet Therapy in Acute Coronary Syndrome

24. Oestreich JH, Smyth SS, Campbell CL. Platelet function analysis: at the edge of

meaning. Thromb Haemost. 2009; 101:217-9.

25. Paré G, Mehta SR, Yusuf S et al. Effects of CYP2C19 genotype on outcomes of clopidogrel treatment. N Engl J Med. 2010; 363:1704-14.

26. Plavix prescribing information. Bridgewater, NJ: Bristol-Myers Squibb/Sanofi Pharmaceuticals Partnership; 2010 Aug. http://products.sanofi-aventis.us/PLAVIX/plavix.pdf (accessed 2010 Nov 3).

27. Price MJ, Endemann S, Gollapudi RR et al. Prognostic significance of post-clopidogrel platelet reactivity assessed by a point-of-care assay on thrombotic events after drug-eluting stent implantation. Eur Heart J. 2008; 29:992-1000.

28. Schömig A. Ticagrelor--is there need for a new player in the antiplatelet-therapy field? N Engl J Med. 2009; 361:1108-11.

29. Serebruany VL, Steinhubl SR, Berger PB et al. Variability in platelet responsiveness to clopidogrel among 544 individuals. J Am Coll Cardiol. 2005; 45:246-51.

30. Shuldiner AR, O’Connell JR, Bliden KP et al. Association of cytochrome P450 2C19 genotype with the antiplatelet effect and clinical efficacy of clopidogrel therapy. JAMA. 2009; 302:849-57.

31. Sibbing D, Braun S, Morath T et al. Platelet reactivity after clopidogrel treatment assessed with point-of-care analysis and early drug-eluting stent thrombosis. J Am Coll Cardiol. 2009; 53:849-56.

32. Sibbing D, Steinhubl SR, Schulz S et al. Platelet aggregation and its association with stent thrombosis and bleeding in clopidogrel-treated patients: initial evidence of a therapeutic window. J Am Coll Cardiol. 2010; 56:317-8.

33. Simon T, Verstuyft C, Mary-Krause M et al. Genetic determinants of response to clopidogrel and cardiovascular events. N Engl J Med. 2009; 360:363-75.

34. Society for Cardiovascular Angiography and Interventions, Society of Thoracic Surgeons, Writing Committee Members et al. ACCF/AHA clopidogrel clinical alert: approaches to the FDA "boxed warning": a report of the American College of Cardiology Foundation Task Force on clinical expert consensus documents and the American Heart Association. Circulation. 2010; 122:537-57.

35. U.S. Food and Drug Administration. FDA reminder to avoid concomitant use of Plavix (clopidogrel) and omeprazole. Updated 10/27/10. www.fda.gov/Drugs/DrugSafety/ucm231161.htm (accessed 2010 Nov 9).

36. Wallentin L, James S, Storey RF et al. Effect of CYP2C19 and ABCB1 single nucleotide polymorphisms on outcomes of treatment with ticagrelor versus clopidogrel for acute coronary syndromes: a genetic substudy of the PLATO trial. Lancet. 2010; 376:1320-8.

37. Xie HG, Kim RB, Wood AJ et al. Molecular basis of ethnic differences in drug disposition and response. Annu Rev Pharmacol Toxicol. 2001; 41:815-50.

51

Page 52: Optimizing Oral Antiplatelet Therapy in Acute Coronary ...Optimizing Oral Antiplatelet Therapy in Acute Coronary Syndrome Toby C. Trujillo, Pharm.D., BCPS (AQ Cardiology) PRESENTATION

Optimizing Oral Antiplatelet Therapy in Acute Coronary Syndrome

S E L F – A S S E S S M E N T Q U E S T I O N S

1. Which of the following laboratory test results would indicate high on-treatment platelet reactivity or clopidogrel nonresponsiveness?

a. PRI 42% (platelet reactivity index) by vasodilator-stimulated phosphoprotein

(VASP) phosphorylation. b. 185 PRU (P2Y12 reaction units) by VerifyNow P2Y12 test. c. 382 arbitrary aggregation units/min in response to adenosine diphosphate (ADP)

by the Multiplate analyzer. d. 55% maximal ADP 5 μM-induced aggregation by light transmission

aggregometry. 2. Which of the following statements most appropriately describes the interaction

between clopidogrel and proton pump inhibitors (PPIs)?

a. Occurs because CYP2C19 is the only enzyme capable of converting clopidogrel to its intermediate and active metabolites.

b. Is supported by the results of the prospective COGENT trial of a fixed-dose combination of clopidogrel and omeprazole.

c. Is relevant because clopidogrel’s prescribing information clearly states that clopidogrel and omeprazole should not be used together.

d. Can be easily evaluated by retrospective studies because PPI use is independent of all other risk factors.

3. According to key cardiology groups, which of the following statements best describes

the role of genetic testing or platelet function testing when selecting antiplatelet therapy? a. Routine testing is recommended, and the results should be considered together

with other risk factors. b. Routine testing is recommended, and the results should be the only risk factor

considered. c. Routine testing is not recommended, but, if available, the results should be

considered together with other risk factors. d. Routine testing is not recommended, and, if available, the results should not be

considered. Answers

1. d 2. c 3. c

52

Page 53: Optimizing Oral Antiplatelet Therapy in Acute Coronary ...Optimizing Oral Antiplatelet Therapy in Acute Coronary Syndrome Toby C. Trujillo, Pharm.D., BCPS (AQ Cardiology) PRESENTATION

Optimizing Oral Antiplatelet Therapy in Acute Coronary Syndrome

Paul P. Dobesh, Pharm.D., FCCP, BCPS (AQ Cardiology) Activity Chair and Moderator Associate Professor of Pharmacy Practice College of Pharmacy University of Nebraska Medical Center Omaha, Nebraska Paul P. Dobesh, Pharm.D., FCCP, BCPS (AQ Cardiology), is Associate Professor of Pharmacy Practice at the University of Nebraska Medical Center in Omaha, Nebraska. Dr. Dobesh earned both his Bachelor of Science in pharmacy and Doctor of Pharmacy degrees from South Dakota State University. He completed a specialty residency in internal medicine at the University of Texas at Austin at Brackenridge Hospital, and he is a board-certified pharmacotherapy specialist with added qualifications in cardiology. Dr. Dobesh’s current responsibilities at the University of Nebraska Medical Center (UNMC) include clinical practice in both internal medicine and cardiology services. He is responsible for teaching pharmacy and medical students, as well as pharmacy and medical residents. His main lecture topics include ischemic heart disease, antithrombotic therapy, and other cardiology and critical care topics. Dr. Dobesh received the UNMC College of Pharmacy Educator of the Year Award for 2010, an award he has received twice within the last four years. Dr. Dobesh has conducted research on antiplatelet and antithrombotic therapy, focusing on the real-world use of these therapies and health-care economics. He has also published book chapters and several manuscripts in this field.

53

Page 54: Optimizing Oral Antiplatelet Therapy in Acute Coronary ...Optimizing Oral Antiplatelet Therapy in Acute Coronary Syndrome Toby C. Trujillo, Pharm.D., BCPS (AQ Cardiology) PRESENTATION

Optimizing Oral Antiplatelet Therapy in Acute Coronary Syndrome

Paul P. Dobesh, Pharm.D., FCCP, BCPS (AQ Cardiology) P R E S E N T A T I O N

Emerging P2Y12 Treatment Strategies O V E R V I E W

Dual antiplatelet therapy (clopidogrel and aspirin) has been part of standard-of-care for the management of patients with acute coronary syndrome and patients undergoing percutaneous coronary intervention for about a decade. Issues, such as a large variability of antiplatelet activity, drug-drug interactions, and pharmacogenomic polymorphisms of metabolism, have raised serious questions about the role of standard dosing of clopidogrel for all patients. A number of alternative approaches have been investigated over the last several years. Increasing the loading and maintenance doses of clopidogrel have been explored with variable success. While this may have some benefit in patients, it is unknown if this strategy will consistently overcome issues of nonresponsiveness. Adding cilostazol has also been investigated, but large phase III trials are still needed to secure this as a solid strategy. The most promise has come in selecting newer P2Y12 antiplatelet agents that do not have the same limitations as clopidogrel. Prasugrel has demonstrated a significant reduction in ischemic endpoints compared with clopidogrel, but at the cost of increased bleeding. Ticagrelor is a different type of P2Y12 inhibitor that does not need to be metabolized to an active metabolite, while it also provides faster reversibility compared with both clopidogrel and prasugrel. Ticagrelor has demonstrated a significant reduction in ischemic endpoints compared with clopidogrel. While there is also an increase in bleeding with ticagrelor, the severity of the bleeding is not as high as with prasugrel. Pharmacists need to understand the pros and cons of these different options as we care for patients with acute coronary syndrome.

54

Page 55: Optimizing Oral Antiplatelet Therapy in Acute Coronary ...Optimizing Oral Antiplatelet Therapy in Acute Coronary Syndrome Toby C. Trujillo, Pharm.D., BCPS (AQ Cardiology) PRESENTATION

Emerging P2Y12 Treatment Strategies

Paul P. Dobesh, Pharm.D., FCCP, BCPS (AQ Cardiology)Associate Professor of Pharmacy Practice

College of PharmacyUniversity of Nebraska Medical Center

Omaha, Nebraska

Emerging Treatment Strategies

• Increasing dose of clopidogrel

• Switching to different P2Y12 inhibitor– Ticlopidine

– Prasugrel– Prasugrel

– Ticagrelor

– Future agents

• Triple antiplatelet therapy– Adding cilostazol

– PAR-1 inhibitors

Table 4: Active Metabolite Pharmacokinetics and Antiplatelet Responses* by CYP2C19 Metabolizer Status

DoseUltrarapid

(n=10)Extensive

(n=10)Intermediate

(n=10)Poor

(n=10)Cmax(ng/mL)

300 mg (24 h) 24 (10) 32 (21) 23 (11) 11 (4)

600 mg (24 h) 36 (13) 44 (27) 39 (23) 17 (6)75 (D 5) 12 (6) 13 (7) 12 (5) 4 (1)

PLAVIX® PI: Clinical PharmacologyAugust 2010

75 mg (Day 5) 12 (6) 13 (7) 12 (5) 4 (1)150 mg (Day 5) 16 (9) 19 (5) 18 (7) 7 (2)

IPA (%)* 300 mg (24 h) 40 (21) 39 (28) 37 (21) 24 (26)600 mg (24 h) 51 (28) 49 (23) 56 (22) 32 (25)75 mg (Day 5) 56 (13) 58 (19) 60 (18) 37 (23)

150 mg (Day 5) 68 (18) 73 (9) 74 (14) 61 (14)VASP-PRI (%)

300 mg (24 h) 73 (12) 68 (16) 78 (12) 91 (12)

600 mg (24 h) 51 (20) 48 (20) 56 (26) 85 (14)75 mg (Day 5) 40 (9) 39 (14) 50 (16) 83 (13)

150 mg (Day 5) 20 (10) 24 (10) 29 (11) 61 (18)

* Reported as mean (SD)

55

Page 56: Optimizing Oral Antiplatelet Therapy in Acute Coronary ...Optimizing Oral Antiplatelet Therapy in Acute Coronary Syndrome Toby C. Trujillo, Pharm.D., BCPS (AQ Cardiology) PRESENTATION

OPTIMUS Study

• Patients with DM type 2 on chronic dual antiplatelet therapy were screened for clopidogrel responsiveness– Platelet aggregation > 50% with 20 µmol/L ADP by

LTA defined nonresponsi enessLTA defined nonresponsiveness

– Screened 64 patients – 40 (62.5%) nonresponsive• Platelet aggregation 39.9% ± 8% in responders

• Platelet aggregation 66.2% ± 8% in nonresponders

– Nonresponders then randomized• Clopidogrel 75 mg daily (n=20)

• Clopidogrel 150 mg daily (n=20)

AngiolilloAngiolillo DJ et al. DJ et al. Circulation. Circulation. 2007; 115:7082007; 115:708--16.16.

OPTIMUS Study

AngiolilloAngiolillo DJ et al. DJ et al. Circulation. Circulation. 2007; 115:7082007; 115:708--16.16.

OPTIMUS Study

AngiolilloAngiolillo DJ et al. DJ et al. Circulation. Circulation. 2007; 115:7082007; 115:708--16.16.

50%

Enlarged slide on page 69

56

Page 57: Optimizing Oral Antiplatelet Therapy in Acute Coronary ...Optimizing Oral Antiplatelet Therapy in Acute Coronary Syndrome Toby C. Trujillo, Pharm.D., BCPS (AQ Cardiology) PRESENTATION

Accelerated Platelet Inhibition by Double Dose Clopidogrel According to Gene Polymorphism

(ACCEL-DOUBLE) Study

• 126 PCI-treated patients

• All given clopidogrel 150 mg daily x 1 month

• Genotyping yp g– CYP3A5

– CYP2C19

– ABCB1

• Platelet function testing performed at 1 month– LTA with 5 and 20 μmol/L

– VerifyNow P2Y12 test

Jeong YH et al. JACC Cardiovasc Interv. 2010; 3:731-41.

ACCEL-DOUBLE: Results

• CYP3A5 and ABCB1 did not influence platelet reactivity

• CYP2C19 predictive of high post-treatment platelet reactivity– OR 5.5 (1.33-23.26) p=0.018

JeongJeong YH et al. YH et al. JACC JACC CardiovascCardiovasc IntervInterv.. 2010; 3:7312010; 3:731--41.41.

Dosing Based on CYP2C19 Status• Patients with high on-treatment platelet reactivity on

clopidogrel 75 mg daily– Identified 41 patients

• 21 with wildtype (*1/*1) CYP2C19 status

• 20 carriers of a 2C19 loss-of-function allele

– VerifyNow P2Y12 test• Baseline (75 mg daily): PRU=285

• Clopidogrel 150 mg daily: PRU=220

• PRU “normalized” (<235) with 150 mg daily in 56% of patients

• No significant difference in antiplatelet response based on 2C19 status

Barker CM et al. JACC Cardiovasc Interv. 2010; 3:1001-7.

Enlarged slide on page 69

57

Page 58: Optimizing Oral Antiplatelet Therapy in Acute Coronary ...Optimizing Oral Antiplatelet Therapy in Acute Coronary Syndrome Toby C. Trujillo, Pharm.D., BCPS (AQ Cardiology) PRESENTATION

Successful PCI with DES without major complication or GP Successful PCI with DES without major complication or GP IIbIIb//IIIaIIIa useuse

PostPost--PCI PCI VerifyNowVerifyNow P2Y12 assay (PRU) 12P2Y12 assay (PRU) 12--24 hours post24 hours post--PCIPCI

PRU ≥ 230PRU ≥ 230??

Nonresponder

Yes Non ~ 6600Responder

Random selection

GRAVITAS Trial

“Standard therapy”clopidogrel 75 mg +

placebo/day

“Standard therapy”clopidogrel 75 mg +

placebo/day

“Tailored therapy”clopidogrel 150 mg/day

Clinical followClinical follow--up and up and VerifyNowVerifyNow assessment at 30 days, 6 monthsassessment at 30 days, 6 months

Primary end point: 6 month CV death, nonfatal MI, ARC Primary end point: 6 month CV death, nonfatal MI, ARC defdef//probprob stent thrombosisstent thrombosis

n=1109 n=586A B Cn=1105

Price M et al. Presented at AHA Scientific Sessions 2010. Abstract 21791.

Ongoing Clinical Trials

ClopidogrelClopidogrelTrialTrial nn PatientsPatients OutcomeOutcome TherapyTherapy

ARCTIC 25002500 Elective Elective 12 month 12 month MD preferenceMD preferencePCI+DESPCI+DES MACEMACE

All trials assessing platelet reactivity by VerifyNow

DANTE 442442 NSTE ACSNSTE ACS 6 & 12 month6 & 12 month 75 mg vs. 150 mg 75 mg vs. 150 mg qdqdPCIPCI MACEMACE

TOPAS-1 450 Previous 6 month 600 mg LD, 75 mg qdPCI ± stent ST

TRIGGER-PCI 21502150 PCIPCI CV death, MICV death, MI 600 mg LD, 75 mg 600 mg LD, 75 mg qdqdvs. vs. prasugrelprasugrel60 mg LD, 10 mg 60 mg LD, 10 mg qdqd

58

GRAVITAS Trial Results

12%

10%

8%

10%

12%

14%High PR - High DoseHigh PR - Standard DoseNormal PR - Standard Dose

p=0.18

Patie

nts

2.3%1.4%

2.3% 2.3%1.4%

0%

2%

4%

6%

CV Death/MI/ST GUSTO Severe/Mod Bleeding

Any GUSTO Bleeding

p=0.10

p=0.98 p=0.20

PR = platelet reactivity PR = platelet reactivity

Price M et al. Presented at AHA Scientific Sessions 2010. Abstract 21791.

% o

f P

Page 59: Optimizing Oral Antiplatelet Therapy in Acute Coronary ...Optimizing Oral Antiplatelet Therapy in Acute Coronary Syndrome Toby C. Trujillo, Pharm.D., BCPS (AQ Cardiology) PRESENTATION

Emerging Treatment Strategies

• Increasing dose of clopidogrel

• Switching to different P2Y12 inhibitor– Ticlopidine

– Prasugrel– Prasugrel

– Ticagrelor

– Future agents

• Triple antiplatelet therapy– Adding cilostazol

– PAR-1 inhibitors

Clopidogrel-to-Ticlopidine Crossover Study

63%

21%

44%34% 33%

50%

75% Absolute Absolute ΔΔ in in AggAggmaxmax ≤ 10%≤ 10%

n=143

% Inhibition of Platelet % Inhibition of Platelet Aggregation < 20%Aggregation < 20%

21%19%

3.5%11.4%

0%

25%

Responders to Both

NR to Clopidogrel

NR to Ticlopidine

NR to Both Responders to Both

NR to Clopidogrel

NR to Ticlopidine

NR to Both

83% of NR to clopidogrel 83% of NR to clopidogrel responded to responded to ticlopidineticlopidine

20% of clopidogrel responders 20% of clopidogrel responders became NR to became NR to ticlopidineticlopidine

67% of NR to clopidogrel 67% of NR to clopidogrel responded to responded to ticlopidineticlopidine

27% of clopidogrel responders 27% of clopidogrel responders became NR to became NR to ticlopidineticlopidine

Campo G et al. Campo G et al. J Am Coll J Am Coll CardiolCardiol. . 2007; 50:11322007; 50:1132--7.7.

1010

1515

HR 0.81HR 0.81(0.73(0.73--0.90)0.90)P=0.0004P=0.0004

Prasugrel Prasugrel

ClopidogrelClopidogrel

oin

t (%

)o

int

(%)

12.112.1

9.99.9

138138eventsevents

TRITON TIMI-38 Trial: Results

CV Death / MI / StrokeCV Death / MI / Stroke

NNT = 46NNT = 46

WiviottWiviott SDSD et al. et al. N N EnglEngl J Med.J Med. 2007; 357:20012007; 357:2001--15.15.

00

55

00 3030 6060 9090 180180 270270 360360 450450

asug easug e

DaysDays

En

dp

oE

nd

po

HR 1.32HR 1.32(1.03(1.03--1.68)1.68)

P=0.03P=0.03

Prasugrel

ClopidogrelClopidogrel1.81.82.42.4

3535eventseventsTIMI Major TIMI Major

NonNon--CABG BleedsCABG Bleeds

NNH = 167NNH = 167

59

Page 60: Optimizing Oral Antiplatelet Therapy in Acute Coronary ...Optimizing Oral Antiplatelet Therapy in Acute Coronary Syndrome Toby C. Trujillo, Pharm.D., BCPS (AQ Cardiology) PRESENTATION

Inhibition of Platelet Aggregation (IPA) at 24 Hours

60.0

80.0

100.0

et A

gg

reg

atio

n (

%)

--20.0

0.0

20.0

40.0

Inh

ibit

ion

of

Pla

tele

Response to Clopidogrel*Responder = *Responder = 25% 25%

IPA at 4 and 24 hIPA at 4 and 24 h

Clopidogrel ResponderClopidogrel Responder

Clopidogrel NonresponderClopidogrel Nonresponder

Response to Prasugrel

Brandt JT et al. Brandt JT et al. Am Heart J.Am Heart J. 2007; 153:66.e92007; 153:66.e9--16.16.

PLATO Study Design

CC

NSTENSTE--ACS (moderateACS (moderate--toto--high risk) or STEMI (if primary PCI)high risk) or STEMI (if primary PCI)ClopidogrelClopidogrel--treated or treated or --naive;naive;

randomized within 24 hours of index event randomized within 24 hours of index event

n=18,624n=18,624

Primary endpoint: CV death + MI + Stroke Primary endpoint: CV death + MI + Stroke Primary safety endpoint: Total major bleedingPrimary safety endpoint: Total major bleeding

ClopidogrelClopidogrelIf preIf pre--treated, no additional loading dose;treated, no additional loading dose;if naive, standard 300 mg loading dose,if naive, standard 300 mg loading dose,

then 75 then 75 mg/day mg/day maintenancemaintenance

TicagrelorTicagrelor180 mg loading dose, then180 mg loading dose, then

90 mg bid maintenance90 mg bid maintenance(additional 90 mg pre(additional 90 mg pre--PCI)PCI)

WallentinWallentin L et al. L et al. N N EnglEngl J Med. J Med. 2009; 361:10452009; 361:1045--57.57.

PLATO Trial: Results

1212

1010

88

66

ve in

cid

ence

(%

)ve

inci

den

ce (

%)

9.89.8

11.711.7Clopidogrel

Ticagrelor

Days after randomizationDays after randomization00 6060 120120 180180 240240 300300 360360

44

22

00Cu

mu

lati

vC

um

ula

tiv

p<0.001HR 0.84 (95% CI 0.77 to 0.92)

NNT=53

WallentinWallentin L et al. L et al. N N EnglEngl J Med. J Med. 2009; 361:10452009; 361:1045--57.57.

60

Page 61: Optimizing Oral Antiplatelet Therapy in Acute Coronary ...Optimizing Oral Antiplatelet Therapy in Acute Coronary Syndrome Toby C. Trujillo, Pharm.D., BCPS (AQ Cardiology) PRESENTATION

PLATO TrialIndividual Endpoints

All patients All patients TicagrelorTicagrelor(n=9,333)(n=9,333)

ClopidogrelClopidogrel(n=9,291)(n=9,291)

HR for HR for (95% CI)(95% CI) p valuep value

Primary objective, (%) Primary objective, (%)

CV death + MI + strokeCV death + MI + stroke 9.89.8 11.711.7 0.84 (0.770.84 (0.77––0.92)0.92) <0.001<0.001

Secondary objectives, (%)Secondary objectives, (%)

Total death + MI + stroke Total death + MI + stroke 10.210.2 12.312.3 0.84 (0.770.84 (0.77––0.92)0.92) <0.001<0.001

CV death + MI + stroke +CV death + MI + stroke +ischemia + TIA + arterial ischemia + TIA + arterial thrombotic events thrombotic events

Myocardial infarctionMyocardial infarction

CV deathCV death

StrokeStroke

14.614.6

5.85.8

4.04.0

1.51.5

16.716.7

6.96.9

5.15.1

1.31.3

0.88 (0.810.88 (0.81––0.95)0.95)

0.84 (0.750.84 (0.75––0.95) 0.95)

0.79 (0.690.79 (0.69––0.91)0.91)

1.17 (0.911.17 (0.91––1.52)1.52)

<0.001<0.001

0.0050.005

0.0010.001

0.220.22

Total deathTotal death 4.54.5 5.95.9 0.78 (0.690.78 (0.69––0.89)0.89) <0.001<0.001

WallentinWallentin L et al. L et al. N N EnglEngl J Med. J Med. 2009; 361:10452009; 361:1045--57.57.

PLATO Results – Major BleedingNon-CABG and CABG Related

4.5%

2 8%

7.4%

5.3%

3.8%

7.9%

5.8%6%

9% Ticagrelor

Clopidogrel

pp=NS=NS

p=NSp=NS

p=0.026p=0.026

p=0.025p=0.0252.8%

2.2%

0%

3%

Non-CABG PLATO

Non-CABG TIMI

CABG PLATO

CABG TIMI

Total PLATO major bleeding: 11.6% Total PLATO major bleeding: 11.6% ticagrelorticagrelor vs. 11.2% clopidogrel; p=0.43vs. 11.2% clopidogrel; p=0.43Total TIMI major bleeding: 7.9% Total TIMI major bleeding: 7.9% ticagrelorticagrelor vs. 7.7% clopidogrel; p=0.57vs. 7.7% clopidogrel; p=0.57

WallentinWallentin L et al. L et al. N N EnglEngl J Med. J Med. 2009; 361:10452009; 361:1045--57.57.

PLATO TrialOther Findings

TicagrelorTicagrelor ClopidogrelClopidogrelAll All PatientsPatients (n=9,235)(n=9,235) (n=9,186)(n=9,186) pp--valuevalue

DyspneaDyspnea, %, %AnyAny 13.813.8 7.87.8 <0.001<0.001With D/C f t d t t tWith D/C f t d t t t 0 90 9 0 10 1 <0 001<0 001

WallentinWallentin L et al. L et al. N N EnglEngl J Med. J Med. 2009; 361:10452009; 361:1045--57.57.

With D/C of study treatmentWith D/C of study treatment 0.90.9 0.10.1 <0.001<0.001

Ventricular pauses, %Ventricular pauses, %First weekFirst week

≥ 3 seconds≥ 3 seconds 5.85.8 3.63.6 0.010.01≥ 5 seconds≥ 5 seconds 2.02.0 1.21.2 0.100.10

At 30 daysAt 30 days≥ 3 seconds≥ 3 seconds 2.12.1 1.71.7 0.520.52≥ 5 seconds ≥ 5 seconds 0.80.8 0.60.6 0.600.60

61

Page 62: Optimizing Oral Antiplatelet Therapy in Acute Coronary ...Optimizing Oral Antiplatelet Therapy in Acute Coronary Syndrome Toby C. Trujillo, Pharm.D., BCPS (AQ Cardiology) PRESENTATION

PLATO TrialOther Findings – Laboratory Parameters

TicagrelorTicagrelor ClopidogrelClopidogrelAll PatientsAll Patients (n=9,325)(n=9,325) (n=9,186)(n=9,186) pp--valuevalue

% increase in % increase in SCrSCr from baselinefrom baselineAt 1 monthAt 1 month 1010 ±± 2222 88 ±± 2121 <0 001<0 001

WallentinWallentin L et al. L et al. N N EnglEngl J Med. J Med. 2009; 361:10452009; 361:1045--5757..

At 1 monthAt 1 month 10 10 ±± 2222 8 8 ±± 2121 <0.001<0.001At 12 monthsAt 12 months 11 11 ±± 2222 9 9 ±± 2222 <0.001<0.001FollowFollow--up visitup visit 10 10 ±± 2222 10 10 ±± 2222 0.590.59

% increase in uric acid from baseline% increase in uric acid from baselineAt 1 monthAt 1 month 14 14 ±± 4646 7 7 ±± 4444 <0.001<0.001At 12 monthsAt 12 months 15 15 ±± 5252 7 7 ±± 3131 <0.001<0.001FollowFollow--up visitup visit 7 7 ±± 4343 8 8 ±± 48 48 0.560.56

The ONSET/OFFSET Study

GurbelGurbel PA et al. PA et al. Circulation.Circulation. 2009; 120:25772009; 120:2577--85.85.

RESPOND Study

• Cross-over study

• Grouped by response to 300 mg LD of Clopidogrel– ≤ 10% absolute Δ in

platelet aggregation to 20

Nonresponders

platelet aggregation to 20 µmol/L ADP by LTA

– Nonresponders (n=41)

– Responders (n=57)

• Clopdiogrel 600 mg LD and 75 mg daily

• Ticagrelor 180 mg LD and 90 mg bid

Gurbel PA et al. Circulation.2010; 121:1188-99.

Responders

Enlarged slide on page 70

62

Page 63: Optimizing Oral Antiplatelet Therapy in Acute Coronary ...Optimizing Oral Antiplatelet Therapy in Acute Coronary Syndrome Toby C. Trujillo, Pharm.D., BCPS (AQ Cardiology) PRESENTATION

What is Next?

• Cangrelor– Parenteral ADP-P2Y12

receptor antagonist• ATP analogue

Pl h lf lif f 5 9

• Elinogrel– First available i.v. and oral

ADP-P2Y12 receptor antagonist• ERASE MI Trial• INNOVATE PCI Trial

• Plasma half-life of 5-9 minutes

• 20 minutes for return to normal platelet function

– CHAMPION Trials• Stopped - no benefit

– PHOENIX Trial

New P2Y12 Inhibitors vs. ClopidogrelMeta-Analysis

Results in patients undergoing any PCI

PrasugrelPrasugrelCangrelorCangrelor

BellemainBellemain--AppaixAppaix A et al. A et al. J Am Coll J Am Coll CardiolCardiol. 2010; 56:1542. 2010; 56:1542--51.51.

TicagrelorElinogrel

PrasugrelPrasugrelCangrelorCangrelorTicagrelor

Elinogrel

New P2Y12 Inhibitors vs. ClopidogrelMeta-Analysis

Results in patients undergoing PCI for STEMI

PrasugrelCangrelor

BellemainBellemain--AppaixAppaix A et al. A et al. J Am Coll J Am Coll CardiolCardiol. 2010; 56:1542. 2010; 56:1542--51.51.

TicagrelorElinogrel

PrasugrelTicagrelor

Elinogrel

Enlarged slide on page 70

Enlarged slide on page 71

63

Page 64: Optimizing Oral Antiplatelet Therapy in Acute Coronary ...Optimizing Oral Antiplatelet Therapy in Acute Coronary Syndrome Toby C. Trujillo, Pharm.D., BCPS (AQ Cardiology) PRESENTATION

Emerging Treatment Strategies

• Increasing dose of clopidogrel

• Switching to different P2Y12 inhibitor– Ticlopidine

– Prasugrel– Prasugrel

– Ticagrelor

– Future agents

• Triple antiplatelet therapy– Adding cilostazol

– PAR-1 inhibitors

Antiplatelet Impact of Cilostazol

AngiolilloAngiolillo DJ et al. DJ et al. EurEur Heart J. Heart J. 2008; 29:22022008; 29:2202--11.11.

CilostazolTriple Antiplatelet Therapy

• OPTIMUS-2– 25 patients with DM type 2 and CAD on DAT

• Placebo x 14 days with cross-over

• Cilostazol 100 mg bid x 14 days with cross-over

– VASP assay (36 3% cilostazol vs 59 9% placebo; p=0 0002)VASP assay (36.3% cilostazol vs. 59.9% placebo; p 0.0002)

• DECREASE Registry– Nonrandomized evaluation of DES patients

• DAT (n=1656) vs. TAT (n=1443) for 12 months– Death: HR 0.76 (0.40 – 1.45); p=0.41

– MI: HR 0.23 (0.08 – 0.70); p=0.001

– Stent thrombosis: HR 0.14 (0.04 – 0.52); p=0.004

– Major bleeding: HR 0.97 (0.44 – 2.12); p=0.94

Angiolillo DJ et al. Eur Heart J. 2008; 29:2202-11.Lee SW et al. Am Heart J. 2010; 159:284-91.e1.

Enlarged slide on page 71

64

Page 65: Optimizing Oral Antiplatelet Therapy in Acute Coronary ...Optimizing Oral Antiplatelet Therapy in Acute Coronary Syndrome Toby C. Trujillo, Pharm.D., BCPS (AQ Cardiology) PRESENTATION

CilostazolTriple Antiplatelet Therapy

• ACCEL-AMI: 90 patients with STEMI after PCI + stenting• Standard group – clopidogrel 75 mg daily

• High maintenance dose group – clopidogrel 150 mg daily

• Triple group – clopidogrel 75 mg daily + cilostazol 100 mg twice daily

– Platelet function testing at baseline and at 1 month

JeongJeong YH et al. YH et al. Circ Circ CardiovascCardiovasc IntervInterv. . 2010; 3:172010; 3:17--26.26.

ADPADP

P2Y12 Inhibition:ClopidogrelTiclodipinePrasugrelTicagrelorElinogrel

Oral Antiplatelet Therapies

P2Y12Thrombin ADP

ADP

Adapted from Schafer AIAdapted from Schafer AI. Am J Med. . Am J Med. 1996; 101:1991996; 101:199--209.209.

CollagenTXA2

TXA2

(Fibrinogen receptor)(Fibrinogen receptor)GP GP IIbIIb//IIIaIIIa

Activation

COX

Aspirin

cAMP

PLATELET

PAR-1PAR-1 Inhibition:VorapaxarAtopaxar

Vorapaxar program(29,500 patients)

Thrombin Receptor AntagonismThrombin Receptor Antagonism

NSTE ACSNSTE ACSn=10,000n=10,000

2º prevention2º preventionn=19,500n=19,500

TRA 2TRA 2°° PP--TIMI 50 trialTIMI 50 trial(Morrow et al. 2009)

TRA*CER trialTRA*CER trial(TRA*CER Committees 2009)

11oo endpoint: Composite of CV endpoint: Composite of CV death, MI, stroke, and urgent death, MI, stroke, and urgent

revascularizationrevascularization

Placebo Placebo

FollowFollow--up 1 up 1 yryr minimumminimum

11oo endpoint: Composite of CV endpoint: Composite of CV death, MI, stroke, urgent death, MI, stroke, urgent

revascularization and revascularization and RI w/ RI w/ rehospitalizationrehospitalization

ClinicalTrials.gov ClinicalTrials.gov Identifier:Identifier: NCT00526474.NCT00526474.

ClinicalTrials.gov ClinicalTrials.gov Identifier:Identifier: NCT00527943.NCT00527943.

VorapaxarVorapaxar

Enlarged slide on page 72

65

Page 66: Optimizing Oral Antiplatelet Therapy in Acute Coronary ...Optimizing Oral Antiplatelet Therapy in Acute Coronary Syndrome Toby C. Trujillo, Pharm.D., BCPS (AQ Cardiology) PRESENTATION

Subjects with NSTE ACS

Randomization within 72 hours of symptom onset

Atopaxar

nn = 603= 603

AtopaxarAtopaxar

Randomize Randomize 1:1:1:11:1:1:1DoubleDouble--blindblind

LANCELOT-ACS Trial

12 Weeks Active Treatment, 4 Weeks Follow-up

Atopaxar400mg LD,50mg/day

Atopaxar400mg LD,200mg/day

Atopaxar400mg LD,100mg/day

Primary Endpoint: Major bleeding (CURE) at 12 weeks

Placebodaily

O’Donoghue M. Presented at TCT 2010, 2010 Sept 23.O’Donoghue M. Presented at TCT 2010, 2010 Sept 23.

8.3%8%

10%

12%

14% TIMI minimal

TIMI minor

TIMI major

LANCELOT-ACS TrialIncidence of any TIMI Bleeding

P trend = 0.63

1.3%2.6%

1.4%0.7%

0.7%

0.7%

1.3%

9.4% 7.3%

7.2% 6.2%

0%

2%

4%

6%

Placebo Active combined atopaxar

50mg daily 100mg daily 200mg daily

n=138 n=455 n=153 n=156 n=146

O’Donoghue M. Presented at TCT 2010, 2010 Sept 23.O’Donoghue M. Presented at TCT 2010, 2010 Sept 23.

5.6% 5.7%6.0%

8.0%

10.0%

12.0%

LANCELOT-ACS TrialIncidence of CV death, MI, or stroke

P trend = 0.28

3.3%

1.9% 2.0%

0.0%

2.0%

4.0%

Placebo Active combined atopaxar

50mg daily 100mg daily 200mg daily

n=142 n=461 n=156 n=157 n=148

O’Donoghue M. Presented at TCT 2010, 2010 Sept 23.O’Donoghue M. Presented at TCT 2010, 2010 Sept 23.

66

Page 67: Optimizing Oral Antiplatelet Therapy in Acute Coronary ...Optimizing Oral Antiplatelet Therapy in Acute Coronary Syndrome Toby C. Trujillo, Pharm.D., BCPS (AQ Cardiology) PRESENTATION

PR is a 78-year-old man (72 kg) with a history of smoking and dyslipidemia. • Just admitted to the hospital with unstable angina• Scheduled to go to PCI tomorrow

Which of the following would be the most appropriate option for providing the optimal reduction in CV death MI and stroke?in CV death, MI, and stroke?a. Clopidogrel 300 mg LD, then 75 mg daily

b. Prasugrel 60 mg LD, then 10 mg daily

c. Ticagrelor 180 mg LD, then 90 mg twice daily

d. Clopidogrel 600 mg LD, then 75 mg daily

TB is a 67-year-old man (80 kg) with history of hypertension, DM type 2, gout, and asthma.• Currently hospitalized with STEMI and going to PCI• Other medical history is non-contributory, except some problems with medication adherence

Which of the following would be the best option for TB?TB?a. Clopidogrel 300 mg LD, then 75 mg daily

b. Prasugrel 60 mg LD, then 10 mg daily

c. Ticagrelor 180 mg LD, then 90 mg twice daily

d. Clopidogrel 300 mg LD, then 75 mg daily and cilostazol 100 mg daily

TC is a 58-year-old man recently started on clopidogrel 75 mg daily. CYP2C19 genotype is CYP2C19*1/*2. VerifyNow test is slightly elevated (PRU = 245). What is the recommended treatment strategy for TC?

a. Continue clopidogrel 75 mg daily

b. Increase clopidogrel maintenance dose to 150 mg daily

c. Start prasugrelc. Start prasugrel

d. Start ticagrelor

67

Page 68: Optimizing Oral Antiplatelet Therapy in Acute Coronary ...Optimizing Oral Antiplatelet Therapy in Acute Coronary Syndrome Toby C. Trujillo, Pharm.D., BCPS (AQ Cardiology) PRESENTATION

Emerging Treatment Strategies

• Increasing dose of clopidogrel

• Switching to different P2Y12 inhibitor– Ticlopidine

– Prasugrel– Prasugrel

– Ticagrelor

– Future agents

• Triple antiplatelet therapy– Adding cilostazol

– PAR-1 inhibitors

68

Page 69: Optimizing Oral Antiplatelet Therapy in Acute Coronary ...Optimizing Oral Antiplatelet Therapy in Acute Coronary Syndrome Toby C. Trujillo, Pharm.D., BCPS (AQ Cardiology) PRESENTATION

OPTIMUS Study

AngiolilloAngiolillo DJ et al. DJ et al. Circulation. Circulation. 2007; 115:7082007; 115:708--16.16.

ACCEL-DOUBLE: Results

• CYP3A5 and ABCB1 did not influence platelet reactivity

• CYP2C19 predictive of high post-treatment platelet reactivityOR 5 5 (1 33 23 26) p=0 018– OR 5.5 (1.33-23.26) p=0.018

JeongJeong YH et al. YH et al. JACC JACC CardiovascCardiovasc IntervInterv.. 2010; 3:7312010; 3:731--41.41.

69

Page 70: Optimizing Oral Antiplatelet Therapy in Acute Coronary ...Optimizing Oral Antiplatelet Therapy in Acute Coronary Syndrome Toby C. Trujillo, Pharm.D., BCPS (AQ Cardiology) PRESENTATION

RESPOND Study

• Cross-over study

• Grouped by response to 300 mg LD of

Nonresponders

300 mg LD of Clopidogrel– ≤ 10% absolute Δ in

platelet aggregation to 20 µmol/L ADP by LTA

– Nonresponders (n=41)

– Responders (n=57)

Responders

p ( )

• Clopdiogrel 600 mg LD and 75 mg daily

• Ticagrelor 180 mg LD and 90 mg bid

Gurbel PA et al. Circulation.2010; 121:1188-99.

New P2Y12 Inhibitors vs. ClopidogrelMeta-Analysis

Results in patients undergoing any PCI

PrasugrelPrasugrelCangrelorCangrelorTicagrelorElinogrel

BellemainBellemain--AppaixAppaix A et al. A et al. J Am Coll J Am Coll CardiolCardiol. 2010; 56:1542. 2010; 56:1542--51.51.

PrasugrelPrasugrelCangrelorCangrelorTicagrelorElinogrel

70

Page 71: Optimizing Oral Antiplatelet Therapy in Acute Coronary ...Optimizing Oral Antiplatelet Therapy in Acute Coronary Syndrome Toby C. Trujillo, Pharm.D., BCPS (AQ Cardiology) PRESENTATION

New P2Y12 Inhibitors vs. ClopidogrelMeta-Analysis

Results in patients undergoing PCI for STEMI

PrasugrelCangrelorTicagrelorElinogrel

BellemainBellemain--AppaixAppaix A et al. A et al. J Am Coll J Am Coll CardiolCardiol. 2010; 56:1542. 2010; 56:1542--51.51.

PrasugrelTicagrelorElinogrel

Antiplatelet Impact of Cilostazol

AngiolilloAngiolillo DJ et al. DJ et al. EurEur Heart J. Heart J. 2008; 29:22022008; 29:2202--11.11.

71

Page 72: Optimizing Oral Antiplatelet Therapy in Acute Coronary ...Optimizing Oral Antiplatelet Therapy in Acute Coronary Syndrome Toby C. Trujillo, Pharm.D., BCPS (AQ Cardiology) PRESENTATION

CilostazolTriple Antiplatelet Therapy

• ACCEL-AMI: 90 patients with STEMI after PCI + stenting• Standard group clopidogrel 75 mg daily• Standard group – clopidogrel 75 mg daily

• High maintenance dose group – clopidogrel 150 mg daily

• Triple group – clopidogrel 75 mg daily + cilostazol 100 mg twice daily

– Platelet function testing at baseline and at 1 month

JeongJeong YH et al. YH et al. Circ Circ CardiovascCardiovasc IntervInterv. . 2010; 3:172010; 3:17--26.26.

72

Page 73: Optimizing Oral Antiplatelet Therapy in Acute Coronary ...Optimizing Oral Antiplatelet Therapy in Acute Coronary Syndrome Toby C. Trujillo, Pharm.D., BCPS (AQ Cardiology) PRESENTATION

Optimizing Oral Antiplatelet Therapy in Acute Coronary Syndrome

S E L E C T E D R E F E R E N C E S 1. Angiolillo DJ, Capranzona P, Goto S et al. A randomized study assessing the impact of

cilostazol on platelet function profiles in patients with diabetes mellitus and coronary artery disease on dual antiplatelet therapy: results of the OPTIMUS-2 study. Eur Heart J. 2008; 29:2202-11.

2. Angiolillo DJ, Shoemaker SB, Desai B et al. Randomized comparison of a high clopidogrel maintenance dose in patients with diabetes mellitus and coronary artery disease: results of the Optimizing Antiplatelet Therapy in Diabetes Mellitus (OPTIMUS) study. Circulation. 2007; 115:708-16.

3. Barker CM, Murray SS, Teirstein PS et al. Pilot study of the antiplatelet effects of increased clopidogrel maintenance dosing and its relationship to CYP2C19 genotype in patients with high on-treatment reactivity. JACC Cardiovasc Interv. 2010; 3:1001-7.

4. Bellemain-Appaix A, Brieger D, Beygui F et al. New P2Y12 inhibitors versus clopidogrel in percutaneous coronary intervention: a meta-analysis. J Am Coll Cardiol. 2010; 56:1542-51.

5. Berger JS, Roe MT, Gibson CM et al. Safety and feasibility of adjunctive antiplatelet therapy with intravenous elinogrel, a direct-acting and reversible P2Y12 ADP-receptor antagonist, before primary percutaneous intervention in patients with ST-elevation myocardial infarction: the Early Rapid ReversAl of platelet thromboSis with intravenous Elinogrel before PCI to optimize reperfusion in acute Myocardial Infarction (ERASE MI) pilot trial. Am Heart J. 2009; 158:998-1004.e1.

6. Bhatt DL, Lincoff AM, Gibson CM et al. for the CHAMPION PLATFORM Investigators. Intravenous platelet blockade with cangrelor during PCI. N Engl J Med. 2009; 361:2330-41.

7. Brandt JT, Payne CD, Wiviott SD et al. A comparison of prasugrel and clopidogrel loading doses on platelet function: magnitude of platelet inhibition is related to active metabolite formation. Am Heart J. 2007; 153:66.e9-16.

8. Campo G, Valgimigli M, Gemmati D et al. Poor responsiveness to clopidogrel: drug-specific or class-effect mechanism? Evidence from a clopidogrel-to-ticlopidine crossover study. J Am Coll Cardiol. 2007; 50:1132-7.

9. Gurbel PA, Bliden KP, Butler K et al. Randomized double-blind assessment of the ONSET and OFFSET of the antiplatelet effects of ticagrelor versus clopidogrel in patients with stable coronary artery disease: the ONSET/OFFSET study. Circulation. 2009; 120:2577-85.

10. Gurbel PA, Bliden KP, Butler K et al. Response to ticagrelor in clopidogrel nonresponders and responders and effect of switching therapies: the RESPOND study. Circulation. 2010; 121:1188-99.

11. Harrington RA, Stone GW, McNulty S et al. Platelet inhibition with cangrelor in patients undergoing PCI. N Engl J Med. 2009; 361:2318-29.

12. Jeong YH, Hwang JY, Kim IS et al. Adding cilostazol to dual antiplatelet therapy achieves greater platelet inhibition than high maintenance dose clopidogrel in patients with acute myocardial infarction: results of the adjunctive cilostazol versus high maintenance dose clopidogrel in patients with AMI (ACCEL-AMI) study. Circ Cardiovasc Interv. 2010; 3:17-26.

73

Page 74: Optimizing Oral Antiplatelet Therapy in Acute Coronary ...Optimizing Oral Antiplatelet Therapy in Acute Coronary Syndrome Toby C. Trujillo, Pharm.D., BCPS (AQ Cardiology) PRESENTATION

Optimizing Oral Antiplatelet Therapy in Acute Coronary Syndrome

13. Jeong YH, Kim IS, Park Y et al. Carriage of cytochrome 2C19 polymorphism is associated

with risk of high post-treatment platelet reactivity on high maintenance-dose clopidogrel of 150 mg/day: results of the ACCEL-DOUBLE (Accelerated Platelet Inhibition by a Double Dose of Clopidogrel According to Gene Polymorphism) study. JACC Cardiovasc Interv. 2010; 3:731-41.

14. Lee SW, Park SW, Yen SC et al. Triple antiplatelet therapy reduces ischemic events after drug-eluting stent implantation: Drug-Eluting stenting followed by Cilostazol treatment REduces Adverse Serious cardiac Events (DECREASE registry). Am Heart J. 2010; 159:284-91.e1.

15. Morrow DA, Scirica BM, Fox KA et al. for the TRA 2°P-TIMI 50 Investigators. Evaluation of a novel antiplatelet agent for secondary prevention in patients with a history of atherosclerotic disease: design and rationale for the Thrombin-Receptor Antagonist in Secondary Prevention of Atherothrombotic Ischemic Events (TRA 2 degrees P)-TIMI 50 trial. Am Heart J. 2009; 158:335-41.e3.

16. O’Donoghue M. LANCELOT ACS: a prospective, randomized, double-blind, placebo-controlled trial of a reversible PAR-1 thrombin receptor antagonist in patients with acute coronary syndromes. Paper presented at TCT 2010, Washington, DC; 2010 Sept 23.

17. Plavix prescribing information. Bridgewater, NJ: Bristol-Myers Squibb/Sanofi Pharmaceuticals Partnership; 2010 Aug. http://products.sanofi-aventis.us/PLAVIX/plavix.pdf (accessed 2010 Nov 3).

18. Price M et al. Standard versus high-dose clopidogrel according to platelet function testing after PCI: results of the GRAVITAS trial. Presented at American Heart Association Scientific Sessions 2010, Chicago; 2010 Nov 16. Abstract 21791.

19. Schafer AI. Antiplatelet therapy. Am J Med. 1996; 101:199-209.

20. TRA*CER Executive and Steering Committees. The Thrombin Receptor Antagonist for Clinical Event Reduction in Acute Coronary Syndrome (TRA*CER) trial: study design and rationale. Am Heart J. 2009; 158:327-34.e4.

21. Wallentin L, Becker RC, Budaj A et al. for the PLATO Investigators. Ticagrelor versus clopidogrel in patients with acute coronary syndromes. N Engl J Med. 2009; 361:1045-57.

22. Wiviott SD, Braunwald E, McCabe CH et al. for the TRITON-TIMI 38 Investigators. Prasugrel versus clopidogrel in patients with acute coronary syndromes. N Engl J Med. 2007; 357:2001-15.

74

Page 75: Optimizing Oral Antiplatelet Therapy in Acute Coronary ...Optimizing Oral Antiplatelet Therapy in Acute Coronary Syndrome Toby C. Trujillo, Pharm.D., BCPS (AQ Cardiology) PRESENTATION

Optimizing Oral Antiplatelet Therapy in Acute Coronary Syndrome

S E L F – A S S E S S M E N T Q U E S T I O N S 1. RJ is a 64-year old man who was recently admitted to the hospital with a non-ST-

segment elevation myocardial infarction. He also has a history of hypertension, diabetes mellitus, and dyslipidemia. He received a loading dose of clopidogrel 300 mg before percutaneous coronary intervention with stent placement, followed by 75 mg daily. On day three of hospitalization, the cardiologist asks for a VerifyNow test. The results demonstrate a platelet reactive units (PRU) value of 300. How would you describe the meaning of this value to the cardiologist?

a. This patient is most likely to have a cardiovascular event. b. This patient has suboptimal platelet inhibition. c. This patient is at increased risk for major bleeding. d. This patient must undergo genetic testing.

2. Which of the following would be an appropriate management strategy for RJ?

a. Give an additional 300 mg loading dose of clopidogrel. b. Give an additional 600 mg loading dose of clopidogrel. c. Give a maintenance dose of clopidogrel 150 mg daily. d. Give a maintenance dose of clopidogrel 300 mg daily.

3. Which of the following outcomes was demonstrated to be significantly reduced by the

use of ticagrelor compared with clopidogrel in the PLATO trial?

a. Stroke. b. Major bleeding. c. Intracranial hemorrhage. d. Cardiovascular death.

4. All of the following provide platelet inhibition via binding to the P2Y12 receptor

EXCEPT

a. Vorapaxar. b. Prasugrel. c. Ticagrelor. d. Elinogrel.

Answers

1. b 2. c 3. d 4. a

75

Page 76: Optimizing Oral Antiplatelet Therapy in Acute Coronary ...Optimizing Oral Antiplatelet Therapy in Acute Coronary Syndrome Toby C. Trujillo, Pharm.D., BCPS (AQ Cardiology) PRESENTATION

Optimizing Oral Antiplatelet Therapy in Acute Coronary Syndrome

L I S T O F A B B R E V I A T I O N S AC adenylyl cyclase

ACC American College of Cardiology

ACCF American College of Cardiology Foundation

ACE angiotensin-converting enzyme

ACS acute coronary syndrome

ADP adenosine diphosphate, endogenous activator of the P2Y12 receptor on platelets

AHA American Heart Association

ARB angiotensin receptor blocker

ARC Academic Research Consortium

ASA aspirin

ATP adenosine triphosphate

ATTC Antithrombotic Trialists’ Collaboration

AUC area under the curve

BMI body mass index

BMS bare-metal stent

Ca calcium

CABG coronary artery bypass grafting

CAD coronary artery disease

cAMP cyclic adenosine monophosphate

CI confidence interval

COX cyclooxegenase

CV cardiovascular

CVD cardiovascular disease

CYP cytochrome P

DAT dual antiplatelet therapy

D/C discontinue

DES drug-eluting stent

DL DerSimonian and Laird

DM diabetes mellitus

DST definite stent thrombosis

ECG electrocardiogram

EM extensive metabolizer

5-HT2A 5-hydroxy tryptamine 2A

GOP gain of function

GP glycoprotein

GUSTO Global Use of Strategies to Open Occluded Coronary Arteries

GWAS genome-wide association study

HDL-C high-density lipoprotein cholesterol

76

Page 77: Optimizing Oral Antiplatelet Therapy in Acute Coronary ...Optimizing Oral Antiplatelet Therapy in Acute Coronary Syndrome Toby C. Trujillo, Pharm.D., BCPS (AQ Cardiology) PRESENTATION

Optimizing Oral Antiplatelet Therapy in Acute Coronary Syndrome

HR hazard ratio

IM intermediate metabolizer

IPA inhibition of platelet aggregation

IP3 inositol triphosphate

LD loading dose

LDL-C low-density lipoprotein cholesterol

LOF loss of function

LTA light transmission aggregometry

MA maximal amplitude (clot strength)

MACE major adverse coronary event

MD physician

MI myocardial infarction

MPA maximal platelet aggregation

NNH number needed to harm

NNT number needed to treat

NR nonresponsive

NSTE non-ST-segment elevation

NSTEMI non-ST-segment elevation myocardial infarction

OR odds ratio

PAR-1 protease-activated receptor 1

PCI percutaneous coronary intervention

PGE1 prostaglandin E1

PI prescribing information

PKA protein kinase

PK/PD pharmacokinetics and pharmacodynamics

PLC phospholipase C

PM poor metabolizer

PPI proton pump inhibitor

PRI platelet reactivity index (unit of measure for VASP phosphorylation assay that measures effectiveness of P2Y12 receptor antagonists)

PRU P2Y12 reaction units (unit of measure for VerifyNow P2Y12 test)

RI recurrent ischemia

ROC receiver operating curve, a graphical plot of true positive rate (sensitivity) versus false positive rate (1- specificity)*

RRR relative risk reduction

SNP single nucleotide polymorphism

ST stent thrombosis

STEMI ST-segment elevation myocardial infarction

TEG thrombelastograph

TIA transient ischemic attack

TIMI Thrombolysis in Myocardial Infarction

77

Page 78: Optimizing Oral Antiplatelet Therapy in Acute Coronary ...Optimizing Oral Antiplatelet Therapy in Acute Coronary Syndrome Toby C. Trujillo, Pharm.D., BCPS (AQ Cardiology) PRESENTATION

Optimizing Oral Antiplatelet Therapy in Acute Coronary Syndrome

TVR target vessel revascularization

TxA2 thromboxane A2

UA unstable angina

UM ultrarapid metabolizer

UTVR urgent target vessel revascularization

VASP vasodilator-stimulated phosphoprotein, a downstream intracellular mediator of P2Y12 receptor activation in platelets

VASP-PRI vasodilator-stimulated phosphoprotein platelet reactivity index

vWF von Willebrand factor

*ROC area under the curve (AUC) of 1.0 would represent a perfectly accurate test, and an AUC of 0.5 represents the line of no discrimination (test is equal to random guess).

78